PrepScholar

Choose Your Test

Sat / act prep online guides and tips, how to write a perfect synthesis essay for the ap language exam.

author image

Advanced Placement (AP)

body-pencil-sharpen-notebook-1

If you're planning to take the AP Language (or AP Lang) exam , you might already know that 55% of your overall exam score will be based on three essays. The first of the three essays you'll have to write on the AP Language exam is called the "synthesis essay." If you want to earn full points on this portion of the AP Lang Exam, you need to know what a synthesis essay is and what skills are assessed by the AP Lang synthesis essay.

In this article, we'll explain the different aspects of the AP Lang synthesis essay, including what skills you need to demonstrate in your synthesis essay response in order to achieve a good score. We'll also give you a full breakdown of a real AP Lang Synthesis Essay prompt, provide an analysis of an AP Lang synthesis essay example, and give you four tips for how to write a synthesis essay.

Let's get started by taking a closer look at how the AP Lang synthesis essay works!

Synthesis Essay AP Lang: What It Is and How It Works

The AP Lang synthesis essay is the first of three essays included in the Free Response section of the AP Lang exam.

The AP Lang synthesis essay portion of the Free Response section lasts for one hour total . This hour consists of a recommended 15 minute reading period and a 40 minute writing period. Keep in mind that these time allotments are merely recommendations, and that exam takers can parse out the allotted 60 minutes to complete the synthesis essay however they choose.

Now, here's what the structure of the AP Lang synthesis essay looks like. The exam presents six to seven sources that are organized around a specific topic (like alternative energy or eminent domain, which are both past synthesis exam topics).

Of these six to seven sources, at least two are visual , including at least one quantitative source (like a graph or pie chart, for example). The remaining four to five sources are print text-based, and each one contains approximately 500 words.

In addition to six to seven sources, the AP Lang exam provides a written prompt that consists of three paragraphs. The prompt will briefly explain the essay topic, then present a claim that students will respond to in an essay that synthesizes material from at least three of the sources provided.

Here's an example prompt provided by the College Board:

Directions : The following prompt is based on the accompanying six sources.

This question requires you to integrate a variety of sources into a coherent, well-written essay. Refer to the sources to support your position; avoid mere paraphrase or summary. Your argument should be central; the sources should support this argument .

Remember to attribute both direct and indirect citations.

Introduction

Television has been influential in United States presidential elections since the 1960's. But just what is this influence, and how has it affected who is elected? Has it made elections fairer and more accessible, or has it moved candidates from pursuing issues to pursuing image?

Read the following sources (including any introductory information) carefully. Then, in an essay that synthesizes at least three of the sources for support, take a position that defends, challenges, or qualifies the claim that television has had a positive impact on presidential elections.

Refer to the sources as Source A, Source B, etc.; titles are included for your convenience.

Source A (Campbell) Source B (Hart and Triece) Source C (Menand) Source D (Chart) Source E (Ranney) Source F (Koppel)

Like we mentioned earlier, this prompt gives you a topic — which it briefly explains — then asks you to take a position. In this case, you'll have to choose a stance on whether television has positively or negatively affected U.S. elections. You're also given six sources to evaluate and use in your response. Now that you have everything you need, now your job is to write an amazing synthesis essay.

But what does "synthesize" mean, exactly? According to the CollegeBoard, when an essay prompt asks you to synthesize, it means that you should "combine different perspectives from sources to form a support of a coherent position" in writing. In other words, a synthesis essay asks you to state your claim on a topic, then highlight the relationships between several sources that support your claim on that topic. Additionally, you'll need to cite specific evidence from your sources to prove your point.

The synthesis essay counts for six of the total points on the AP Lang exam . Students can receive 0-1 points for writing a thesis statement in the essay, 0-4 based on incorporation of evidence and commentary, and 0-1 points based on sophistication of thought and demonstrated complex understanding of the topic.

You'll be evaluated based on how effectively you do the following in your AP Lang synthesis essay:

Write a thesis that responds to the exam prompt with a defensible position

Provide specific evidence that to support all claims in your line of reasoning from at least three of the sources provided, and clearly and consistently explain how the evidence you include supports your line of reasoning

Demonstrate sophistication of thought by either crafting a thoughtful argument, situating the argument in a broader context, explaining the limitations of an argument

Make rhetorical choices that strengthen your argument and/or employ a vivid and persuasive style throughout your essay.

If your synthesis essay meets the criteria above, then there's a good chance you'll score well on this portion of the AP Lang exam!

If you're looking for even more information on scoring, the College Board has posted the AP Lang Free Response grading rubric on its website. ( You can find it here. ) We recommend taking a close look at it since it includes additional details about the synthesis essay scoring.

body-chisel-break-apart

Don't be intimidated...we're going to teach you how to break down even the hardest AP synthesis essay prompt.

Full Breakdown of a Real AP Lang Synthesis Essay Prompt

In this section, we'll teach you how to analyze and respond to a synthesis essay prompt in five easy steps, including suggested time frames for each step of the process.

Step 1: Analyze the Prompt

The very first thing to do when the clock starts running is read and analyze the prompt. To demonstrate how to do this, we'll look at the sample AP Lang synthesis essay prompt below. This prompt comes straight from the 2018 AP Lang exam:

Eminent domain is the power governments have to acquire property from private owners for public use. The rationale behind eminent domain is that governments have greater legal authority over lands within their dominion than do private owners. Eminent domain has been instituted in one way or another throughout the world for hundreds of years.

Carefully read the following six sources, including the introductory information for each source. Then synthesize material from at least three of the sources and incorporate it into a coherent, well-developed essay that defends, challenges, or qualifies the notion that eminent domain is productive and beneficial.

Your argument should be the focus of your essay. Use the sources to develop your argument and explain the reasoning for it. Avoid merely summarizing the sources. Indicate clearly which sources you are drawing from, whether through direct quotation, paraphrase, or summary. You may cite the sources as Source A, Source B, etc., or by using the descriptions in parentheses.

On first read, you might be nervous about how to answer this prompt...especially if you don't know what eminent domain is! But if you break the prompt down into chunks, you'll be able to figure out what the prompt is asking you to do in no time flat.

To get a full understanding of what this prompt wants you to do, you need to identify the most important details in this prompt, paragraph by paragraph. Here's what each paragraph is asking you to do:

  • Paragraph 1: The prompt presents and briefly explains the topic that you'll be writing your synthesis essay about. That topic is the concept of eminent domain.
  • Paragraph 2: The prompt presents a specific claim about the concept of eminent domain in this paragraph: Eminent domain is productive and beneficial. This paragraph instructs you to decide whether you want to defend, challenge, or qualify that claim in your synthesis essay , and use material from at least three of the sources provided in order to do so.
  • Paragraph 3: In the last paragraph of the prompt, the exam gives you clear instructions about how to approach writing your synthesis essay . First, make your argument the focus of the essay. Second, use material from at least three of the sources to develop and explain your argument. Third, provide commentary on the material you include, and provide proper citations when you incorporate quotations, paraphrases, or summaries from the sources provided.

So basically, you'll have to agree with, disagree with, or qualify the claim stated in the prompt, then use at least three sources substantiate your answer. Since you probably don't know much about eminent domain, you'll probably decide on your position after you read the provided sources.

To make good use of your time on the exam, you should spend around 2 minutes reading the prompt and making note of what it's asking you to do. That will leave you plenty of time to read the sources provided, which is the next step to writing a synthesis essay.

Step 2: Read the Sources Carefully

After you closely read the prompt and make note of the most important details, you need to read all of the sources provided. It's tempting to skip one or two sources to save time--but we recommend you don't do this. That's because you'll need a thorough understanding of the topic before you can accurately address the prompt!

For the sample exam prompt included above, there are six sources provided. We're not going to include all of the sources in this article, but you can view the six sources from this question on the 2018 AP Lang exam here . The sources include five print-text sources and one visual source, which is a cartoon.

As you read the sources, it's important to read quickly and carefully. Don't rush! Keep your pencil in hand to quickly mark important passages that you might want to use as evidence in your synthesis. While you're reading the sources and marking passages, you want to think about how the information you're reading influences your stance on the issue (in this case, eminent domain).

When you finish reading, take a few seconds to summarize, in a phrase or sentence, whether the source defends, challenges, or qualifies whether eminent domain is beneficial (which is the claim in the prompt) . Though it might not feel like you have time for this, it's important to give yourself these notes about each source so you know how you can use each one as evidence in your essay.

Here's what we mean: say you want to challenge the idea that eminent domain is useful. If you've jotted down notes about each source and what it's saying, it will be easier for you to pull the relevant information into your outline and your essay.

So how much time should you spend reading the provided sources? The AP Lang exam recommends taking 15 minutes to read the sources . If you spend around two of those minutes reading and breaking down the essay prompt, it makes sense to spend the remaining 13 minutes reading and annotating the sources.

If you finish reading and annotating early, you can always move on to drafting your synthesis essay. But make sure you're taking your time and reading carefully! It's better to use a little extra time reading and understanding the sources now so that you don't have to go back and re-read the sources later.

body-weightlifting-lift-strong

A strong thesis will do a lot of heavy lifting in your essay. (See what we did there?)

Step 3: Write a Strong Thesis Statement

After you've analyzed the prompt and thoroughly read the sources, the next thing you need to do in order to write a good synthesis essay is write a strong thesis statement .

The great news about writing a thesis statement for this synthesis essay is that you have all the tools you need to do it at your fingertips. All you have to do in order to write your thesis statement is decide what your stance is in relationship to the topic provided.

In the example prompt provided earlier, you're essentially given three choices for how to frame your thesis statement: you can either defend, challenge, or qualify a claim that's been provided by the prompt, that eminent domain is productive and beneficial . Here's what that means for each option:

If you choose to defend the claim, your job will be to prove that the claim is correct . In this case, you'll have to show that eminent domain is a good thing.

If you choose to challenge the claim, you'll argue that the claim is incorrect. In other words, you'll argue that eminent domain isn't productive or beneficial.

If you choose to qualify, that means you'll agree with part of the claim, but disagree with another part of the claim. For instance, you may argue that eminent domain can be a productive tool for governments, but it's not beneficial for property owners. Or maybe you argue that eminent domain is useful in certain circumstances, but not in others.

When you decide whether you want your synthesis essay to defend, challenge, or qualify that claim, you need to convey that stance clearly in your thesis statement. You want to avoid simply restating the claim provided in the prompt, summarizing the issue without making a coherent claim, or writing a thesis that doesn't respond to the prompt.

Here's an example of a thesis statement that received full points on the eminent domain synthesis essay:

Although eminent domain can be misused to benefit private interests at the expense of citizens, it is a vital tool of any government that intends to have any influence on the land it governs beyond that of written law.

This thesis statement received full points because it states a defensible position and establishes a line of reasoning on the issue of eminent domain. It states the author's position (that some parts of eminent domain are good, but others are bad), then goes on to explain why the author thinks that (it's good because it allows the government to do its job, but it's bad because the government can misuse its power.)

Because this example thesis statement states a defensible position and establishes a line of reasoning, it can be elaborated upon in the body of the essay through sub-claims, supporting evidence, and commentary. And a solid argument is key to getting a six on your synthesis essay for AP Lang!

Looking for help studying for your AP exam?

Our one-on-one online AP tutoring services can help you prepare for your AP exams. Get matched with a top tutor who got a high score on the exam you're studying for!

Get a 5 On Your AP Exam

Step 4: Create a Bare-Bones Essay Outline

Once you've got your thesis statement drafted, you have the foundation you need to develop a bare bones outline for your synthesis essay. Developing an outline might seem like it's a waste of your precious time, but if you develop your outline well, it will actually save you time when you start writing your essay.

With that in mind, we recommend spending 5 to 10 minutes outlining your synthesis essay . If you use a bare-bones outline like the one below, labeling each piece of content that you need to include in your essay draft, you should be able to develop out the most important pieces of the synthesis before you even draft the actual essay.

To help you see how this can work on test day, we've created a sample outline for you. You can even memorize this outline to help you out on test day! In the outline below, you'll find places to fill in a thesis statement, body paragraph topic sentences, evidence from the sources provided, and commentary :

  • Present the context surrounding the essay topic in a couple of sentences (this is a good place to use what you learned about the major opinions or controversies about the topic from reading your sources).
  • Write a straightforward, clear, and concise thesis statement that presents your stance on the topic
  • Topic sentence presenting first supporting point or claim
  • Evidence #1
  • Commentary on Evidence #1
  • Evidence #2 (if needed)
  • Commentary on Evidence #2 (if needed)
  • Topic sentence presenting second supporting point or claim
  • Topic sentence presenting three supporting point or claim
  • Sums up the main line of reasoning that you developed and defended throughout the essay
  • Reiterates the thesis statement

Taking the time to develop these crucial pieces of the synthesis in a bare-bones outline will give you a map for your final essay. Once you have a map, writing the essay will be much easier.

Step 5: Draft Your Essay Response

The great thing about taking a few minutes to develop an outline is that you can develop it out into your essay draft. After you take about 5 to 10 minutes to outline your synthesis essay, you can use the remaining 30 to 35 minutes to draft your essay and review it.

Since you'll outline your essay before you start drafting, writing the essay should be pretty straightforward. You'll already know how many paragraphs you're going to write, what the topic of each paragraph will be, and what quotations, paraphrases, or summaries you're going to include in each paragraph from the sources provided. You'll just have to fill in one of the most important parts of your synthesis—your commentary.

Commentaries are your explanation of why your evidence supports the argument you've outlined in your thesis. Your commentary is where you actually make your argument, which is why it's such a critical part of your synthesis essay.

When thinking about what to say in your commentary, remember one thing the AP Lang synthesis essay prompt specifies: don't just summarize the sources. Instead, as you provide commentary on the evidence you incorporate, you need to explain how that evidence supports or undermines your thesis statement . You should include commentary that offers a thoughtful or novel perspective on the evidence from your sources to develop your argument.

One very important thing to remember as you draft out your essay is to cite your sources. The AP Lang exam synthesis essay prompt indicates that you can use generic labels for the sources provided (e.g. "Source 1," "Source 2," "Source 3," etc.). The exam prompt will indicate which label corresponds with which source, so you'll need to make sure you pay attention and cite sources accurately. You can cite your sources in the sentence where you introduce a quote, summary, or paraphrase, or you can use a parenthetical citation. Citing your sources affects your score on the synthesis essay, so remembering to do this is important.

body-green-arrow-down

Keep reading for a real-life example of a great AP synthesis essay response!

Real-Life AP Synthesis Essay Example and Analysis

If you're still wondering how to write a synthesis essay, examples of real essays from past AP Lang exams can make things clearer. These real-life student AP synthesis essay responses can be great for helping you understand how to write a synthesis essay that will knock the graders' socks off .

While there are multiple essay examples online, we've chosen one to take a closer look at. We're going to give you a brief analysis of one of these example student synthesis essays from the 2019 AP Lang Exam below!

Example Synthesis Essay AP Lang Response

To get started, let's look at the official prompt for the 2019 synthesis essay:

In response to our society's increasing demand for energy, large-scale wind power has drawn attention from governments and consumers as a potential alternative to traditional materials that fuel our power grids, such as coal, oil, natural gas, water, or even newer sources such as nuclear or solar power. Yet the establishment of large-scale, commercial-grade wind farms is often the subject of controversy for a variety of reasons.

Carefully read the six sources, found on the AP English Language and Composition 2019 Exam (Question 1), including the introductory information for each source. Write an essay that synthesizes material from at least three of the sources and develops your position on the most important factors that an individual or agency should consider when deciding whether to establish a wind farm.

Source A (photo) Source B (Layton) Source C (Seltenrich) Source D (Brown) Source E (Rule) Source F (Molla)

In your response you should do the following:

  • Respond to the prompt with a thesis presents a defensible position.
  • Select and use evidence from at least 3 of the provided sources to support your line of reasoning. Indicate clearly the sources used through direct quotation, paraphrase, or summary. Sources may be cited as Source A, Source B, etc., or by using the description in parentheses.
  • Explain how the evidence supports your line of reasoning.
  • Use appropriate grammar and punctuation in communicating your argument.

Now that you know exactly what the prompt asked students to do on the 2019 AP Lang synthesis essay, here's an AP Lang synthesis essay example, written by a real student on the AP Lang exam in 2019:

[1] The situation has been known for years, and still very little is being done: alternative power is the only way to reliably power the changing world. The draw of power coming from industry and private life is overwhelming current sources of non-renewable power, and with dwindling supplies of fossil fuels, it is merely a matter of time before coal and gas fuel plants are no longer in operation. So one viable alternative is wind power. But as with all things, there are pros and cons. The main factors for power companies to consider when building wind farms are environmental boon, aesthetic, and economic factors.

[2] The environmental benefits of using wind power are well-known and proven. Wind power is, as qualified by Source B, undeniably clean and renewable. From their production requiring very little in the way of dangerous materials to their lack of fuel, besides that which occurs naturally, wind power is by far one of the least environmentally impactful sources of power available. In addition, wind power by way of gearbox and advanced blade materials, has the highest percentage of energy retention. According to Source F, wind power retains 1,164% of the energy put into the system – meaning that it increases the energy converted from fuel (wind) to electricity 10 times! No other method of electricity production is even half that efficient. The efficiency and clean nature of wind power are important to consider, especially because they contribute back to power companies economically.

[3] Economically, wind power is both a boon and a bone to electric companies and other users. For consumers, wind power is very cheap, leading to lower bills than from any other source. Consumers also get an indirect reimbursement by way of taxes (Source D). In one Texan town, McCamey, tax revenue increased 30% from a wind farm being erected in the town. This helps to finance improvements to the town. But, there is no doubt that wind power is also hurting the power companies. Although, as renewable power goes, wind is incredibly cheap, it is still significantly more expensive than fossil fuels. So, while it is helping to cut down on emissions, it costs electric companies more than traditional fossil fuel plants. While the general economic trend is positive, there are some setbacks which must be overcome before wind power can take over as truly more effective than fossil fuels.

[4] Aesthetics may be the greatest setback for power companies. Although there may be significant economic and environmental benefit to wind power, people will always fight to preserve pure, unspoiled land. Unfortunately, not much can be done to improve the visual aesthetics of the turbines. White paint is the most common choice because it "[is] associated with cleanliness." (Source E). But, this can make it stand out like a sore thumb, and make the gargantuan machines seem more out of place. The site can also not be altered because it affects generating capacity. Sound is almost worse of a concern because it interrupts personal productivity by interrupting people's sleep patterns. One thing for power companies to consider is working with turbine manufacturing to make the machines less aesthetically impactful, so as to garner greater public support.

[5] As with most things, wind power has no easy answer. It is the responsibility of the companies building them to weigh the benefits and the consequences. But, by balancing economics, efficiency, and aesthetics, power companies can create a solution which balances human impact with environmental preservation.

And that's an entire AP Lang synthesis essay example, written in response to a real AP Lang exam prompt! It's important to remember AP Lang exam synthesis essay prompts are always similarly structured and worded, and students often respond in around the same number of paragraphs as what you see in the example essay response above.

Next, let's analyze this example essay and talk about what it does effectively, where it could be improved upon, and what score past exam scorers awarded it.

To get started on an analysis of the sample synthesis essay, let's look at the scoring commentary provided by the College Board:

  • For development of thesis, the essay received 1 out of 1 possible points
  • For evidence and commentary, the essay received 4 out of 4 possible points
  • For sophistication of thought, the essay received 0 out of 1 possible points.

This means that the final score for this example essay was a 5 out of 6 possible points . Let's look more closely at the content of the example essay to figure out why it received this score breakdown.

Thesis Development

The thesis statement is one of the three main categories that is taken into consideration when you're awarded points on this portion of the exam. This sample essay received 1 out of 1 total points.

Now, here's why: the thesis statement clearly and concisely conveys a position on the topic presented in the prompt--alternative energy and wind power--and defines the most important factors that power companies should consider when deciding whether to establish a wind farm.

Evidence and Commentary

The second key category taken into consideration when synthesis exams are evaluated is incorporation of evidence and commentary. This sample received 4 out of 4 possible points for this portion of the synthesis essay. At bare minimum, this sample essay meets the requirement mentioned in the prompt that the writer incorporate evidence from at least three of the sources provided.

On top of that, the writer does a good job of connecting the incorporated evidence back to the claim made in the thesis statement through effective commentary. The commentary in this sample essay is effective because it goes beyond just summarizing what the provided sources say. Instead, it explains and analyzes the evidence presented in the selected sources and connects them back to supporting points the writer makes in each body paragraph.

Finally, the writer of the essay also received points for evidence and commentary because the writer developed and supported a consistent line of reasoning throughout the essay . This line of reasoning is summed up in the fourth paragraph in the following sentence: "One thing for power companies to consider is working with turbine manufacturing to make the machines less aesthetically impactful, so as to garner greater public support."

Because the writer did a good job consistently developing their argument and incorporating evidence, they received full marks in this category. So far, so good!

Sophistication of Thought

Now, we know that this essay received a score of 5 out of 6 total points, and the place where the writer lost a point was on the basis of sophistication of thought, for which the writer received 0 out of 1 points. That's because this sample essay makes several generalizations and vague claims where it could have instead made specific claims that support a more balanced argument.

For example, in the following sentence from the 5th paragraph of the sample essay, the writer misses the opportunity to state specific possibilities that power companies should consider for wind energy . Instead, the writer is ambiguous and non-committal, saying, "As with most things, wind power has no easy answer. It is the responsibility of the companies building them to weigh the benefits and consequences."

If the writer of this essay was interested in trying to get that 6th point on the synthesis essay response, they could consider making more specific claims. For instance, they could state the specific benefits and consequences power companies should consider when deciding whether to establish a wind farm. These could include things like environmental impacts, economic impacts, or even population density!

Despite losing one point in the last category, this example synthesis essay is a strong one. It's well-developed, thoughtfully written, and advances an argument on the exam topic using evidence and support throughout.

body-number-four-post-it-note

4 Tips for How to Write a Synthesis Essay

AP Lang is a timed exam, so you have to pick and choose what you want to focus on in the limited time you're given to write the synthesis essay. Keep reading to get our expert advice on what you should focus on during your exam.

Tip 1: Read the Prompt First

It may sound obvious, but when you're pressed for time, it's easy to get flustered. Just remember: when it comes time to write the synthesis essay, read the prompt first !

Why is it so important to read the prompt before you read the sources? Because when you're aware of what kind of question you're trying to answer, you'll be able to read the sources more strategically. The prompt will help give you a sense of what claims, points, facts, or opinions to be looking for as you read the sources.

Reading the sources without having read the prompt first is kind of like trying to drive while wearing a blindfold: you can probably do it, but it's likely not going to end well!

Tip 2: Make Notes While You Read

During the 15-minute reading period at the beginning of the synthesis essay, you'll be reading through the sources as quickly as you can. After all, you're probably anxious to start writing!

While it's definitely important to make good use of your time, it's also important to read closely enough that you understand your sources. Careful reading will allow you to identify parts of the sources that will help you support your thesis statement in your essay, too.

As you read the sources, consider marking helpful passages with a star or check mark in the margins of the exam so you know which parts of the text to quickly re-read as you form your synthesis essay. You might also consider summing up the key points or position of each source in a sentence or a few words when you finish reading each source during the reading period. Doing so will help you know where each source stands on the topic given and help you pick the three (or more!) that will bolster your synthesis argument.

Tip 3: Start With the Thesis Statement

If you don't start your synthesis essay with a strong thesis statement, it's going to be tough to write an effective synthesis essay. As soon as you finish reading and annotating the provided sources, the thing you want to do next is write a strong thesis statement.

According to the CollegeBoard grading guidelines for the AP Lang synthesis essay, a strong thesis statement will respond to the prompt— not restate or rephrase the prompt. A good thesis will take a clear, defensible position on the topic presented in the prompt and the sources.

In other words, to write a solid thesis statement to guide the rest of your synthesis essay, you need to think about your position on the topic at hand and then make a claim about the topic based on your position. This position will either be defending, challenging, or qualifying the claim made in the essay's prompt.

The defensible position that you establish in your thesis statement will guide your argument in the rest of the essay, so it's important to do this first. Once you have a strong thesis statement, you can begin outlining your essay.

Tip 4: Focus on Your Commentary

Writing thoughtful, original commentary that explains your argument and your sources is important. In fact, doing this well will earn you four points (out of a total of six)!

AP Lang provides six to seven sources for you on the exam, and you'll be expected to incorporate quotations, paraphrases, or summaries from at least three of those sources into your synthesis essay and interpret that evidence for the reader.

While incorporating evidence is very important, in order to get the extra point for "sophistication of thought" on the synthesis essay, it's important to spend more time thinking about your commentary on the evidence you choose to incorporate. The commentary is your chance to show original thinking, strong rhetorical skills, and clearly explain how the evidence you've included supports the stance you laid out in your thesis statement.

To earn the 6th possible point on the synthesis essay, make sure your commentary demonstrates a nuanced understanding of the source material, explains this nuanced understanding, and places the evidence incorporated from the sources in conversation with each other. To do this, make sure you're avoiding vague language. Be specific when you can, and always tie your commentary back to your thesis!

body-person-arrows-next

What's Next?

There's a lot more to the AP Language exam than just the synthesis essay. Be sure to check out our expert guide to the entire exam , then learn more about the tricky multiple choice section .

Is the AP Lang exam hard...or is it easy? See how it stacks up to other AP tests on our list of the hardest AP exams .

Did you know there are technically two English AP exams? You can learn more about the second English AP test, the AP Literature exam, in this article . And if you're confused about whether you should take the AP Lang or AP Lit test , we can help you make that decision, too.

Want to improve your SAT score by 160 points or your ACT score by 4 points? We've written a guide for each test about the top 5 strategies you must be using to have a shot at improving your score. Download it for free now:

Get eBook: 5 Tips for 160+ Points

Ashley Sufflé Robinson has a Ph.D. in 19th Century English Literature. As a content writer for PrepScholar, Ashley is passionate about giving college-bound students the in-depth information they need to get into the school of their dreams.

Student and Parent Forum

Our new student and parent forum, at ExpertHub.PrepScholar.com , allow you to interact with your peers and the PrepScholar staff. See how other students and parents are navigating high school, college, and the college admissions process. Ask questions; get answers.

Join the Conversation

Ask a Question Below

Have any questions about this article or other topics? Ask below and we'll reply!

Improve With Our Famous Guides

  • For All Students

The 5 Strategies You Must Be Using to Improve 160+ SAT Points

How to Get a Perfect 1600, by a Perfect Scorer

Series: How to Get 800 on Each SAT Section:

Score 800 on SAT Math

Score 800 on SAT Reading

Score 800 on SAT Writing

Series: How to Get to 600 on Each SAT Section:

Score 600 on SAT Math

Score 600 on SAT Reading

Score 600 on SAT Writing

Free Complete Official SAT Practice Tests

What SAT Target Score Should You Be Aiming For?

15 Strategies to Improve Your SAT Essay

The 5 Strategies You Must Be Using to Improve 4+ ACT Points

How to Get a Perfect 36 ACT, by a Perfect Scorer

Series: How to Get 36 on Each ACT Section:

36 on ACT English

36 on ACT Math

36 on ACT Reading

36 on ACT Science

Series: How to Get to 24 on Each ACT Section:

24 on ACT English

24 on ACT Math

24 on ACT Reading

24 on ACT Science

What ACT target score should you be aiming for?

ACT Vocabulary You Must Know

ACT Writing: 15 Tips to Raise Your Essay Score

How to Get Into Harvard and the Ivy League

How to Get a Perfect 4.0 GPA

How to Write an Amazing College Essay

What Exactly Are Colleges Looking For?

Is the ACT easier than the SAT? A Comprehensive Guide

Should you retake your SAT or ACT?

When should you take the SAT or ACT?

Stay Informed

ap lang synthesis essay monolingual

Get the latest articles and test prep tips!

Looking for Graduate School Test Prep?

Check out our top-rated graduate blogs here:

GRE Online Prep Blog

GMAT Online Prep Blog

TOEFL Online Prep Blog

Holly R. "I am absolutely overjoyed and cannot thank you enough for helping me!”

What are your chances of acceptance?

Calculate for all schools, your chance of acceptance.

Duke University

Your chancing factors

Extracurriculars.

ap lang synthesis essay monolingual

How to Write the AP Lang Synthesis Essay + Example

Do you know how to improve your profile for college applications.

See how your profile ranks among thousands of other students using CollegeVine. Calculate your chances at your dream schools and learn what areas you need to improve right now — it only takes 3 minutes and it's 100% free.

Show me what areas I need to improve

What’s Covered:

What is the ap lang synthesis essay, how will ap scores affect my college chances.

AP English Language and Composition, commonly known as AP Lang, is one of the most engaging and popular AP classes offered at most high schools, with over 535,000 students taking the class . AP Lang tests your ability to analyze written pieces, synthesize information, write rhetorical essays, and create cohesive and concrete arguments. However, the class is rather challenging as only 62% of students were able to score a three or higher on the exam. 

The AP Lang exam has two sections. The first consists of 45 multiple choice questions which need to be completed in an hour. This portion counts for around 45% of your total score. These questions ask students to analyze written pieces and answer questions related to each respective passage.  All possible answer choices can be found within the text, and no prior knowledge of literature is needed to understand the passages.

The second section contains three free-response questions to be finished in under two hours and 15 minutes. This section counts for 55% of your score and includes the synthesis essay, the rhetorical essay, and the argumentative essay.

  • The synthesis essay requires you to read 6-7 sources and create an argument using at least three sources.
  • The rhetorical analysis essay requires you to describe how a piece of writing evokes specific meanings and symbolism.
  • The argumentative essay requires you to pick a perspective of a debate and create an argument based on the evidence provided.

In this post, we will take a look at the AP Lang synthesis essay and discuss tips and tricks to master this part of the exam. We will also provide an example of a well-written essay for review.  

The AP Lang synthesis essay is the first of three essays included in the Free Response section of the AP Lang exam. The exam presents 6-7 sources that are organized around a specific topic, with two of those sources purely visual, including a single quantitative source (like a graph or pie chart). The remaining 4-5 sources are text-based, containing around 500 words each. It’s recommended that students spend an hour on this essay—15 minute reading period, 40 minutes writing, and 5 minutes of spare time to check over work.

Each synthesis essay has a topic that all the sources will relate to. A prompt will explaining the topic and provide some background, although the topics are usually broad so you will probably know something related to the issue. It will also present a claim that students will respond to in an essay format using information from at least three of the provided sources. You will need to take a stance, either agreeing or disagreeing with the position provided in the claim. 

According to the CollegeBoard, they are looking for essays that “combine different perspectives from sources to form a support of a coherent position.” This means that you must state your claim on the topic and highlight relationships between several sources that support your specific position on the topic. Additionally, you’ll need to cite clear evidence from your sources to prove your point.

The synthesis essay counts for six points on the AP Lang exam. Students can receive 0-1 points for writing a thesis statement, 0-4 based on the incorporation of evidence and commentary, and 0-1 points based on the sophistication of thought and demonstration of complex understanding.

While this essay seems extremely overwhelming, considering there are a total of three free-response essays to complete, with proper time management and practiced skills, this essay is manageable and straightforward. In order to enhance the time management aspect of the test to the best of your ability, it is essential to divide the essay up into five key steps.

Step 1: Analyze the Prompt

As soon as the clock starts, carefully read and analyze what the prompt asks from you. It might be helpful to markup the text to identify the most critical details. You should only spend around 2 minutes reading the prompt so you have enough time to read all the sources and figure out your argument. Don’t feel like you need to immediately pick your stance on the claim right after reading the prompt. You should read the sources before you commit to your argument.

Step 2: Read the Sources Carefully

Although you are only required to use 3 of the 6-7 sources provides, make sure you read ALL of the sources. This will allow you to better understand the topic and make the most educated decision of which sources to use in your essay. Since there are a lot of sources to get through, you will need to read quickly and carefully.

Annotating will be your best friend during the reading period. Highlight and mark important concepts or lines from each passage that would be helpful in your essay. Your argument will probably begin forming in your head as you go through the passages, so you will save yourself a lot of time later on if you take a few seconds to write down notes in the margins. After you’ve finished reading a source, reflect on whether the source defends, challenges, or qualifies your argument.

You will have around 13 minutes to read through all the sources, but it’s very possible you will finish earlier if you are a fast reader. Take the leftover time to start developing your thesis and organizing your thoughts into an outline so you have more time to write. 

Step 3: Write a Strong Thesis Statement 

In order to write a good thesis statement, all you have to do is decide your stance on the claim provided in the prompt and give an overview of your evidence. You essentially have three choices on how to frame your thesis statement: You can defend, challenge or qualify a claim that’s been provided by the prompt. 

  • If you are defending the claim, your job will be to prove that the claim is correct .
  • If you are challenging the claim, your job will be to prove that the claim is incorrect .
  • If you choose to qualify the claim, your job will be to agree to a part of the claim and disagree with another part of the claim. 

A strong thesis statement will clearly state your stance without summarizing the issue or regurgitating the claim. The CollegeBoard is looking for a thesis statement that “states a defensible position and establishes a line of reasoning on the issue provided in the prompt.”

Step 4: Create a Minimal Essay Outline

Developing an outline might seem like a waste of time when you are up against the clock, but believe us, taking 5-10 minutes to outline your essay will be much more useful in the long run than jumping right into the essay.

Your outline should include your thesis statement and three main pieces of evidence that will constitute each body paragraph. Under each piece of evidence should be 2-3 details from the sources that you will use to back up your claim and some commentary on how that evidence proves your thesis.

Step 5: Write your Essay

Use the remaining 30-35 minutes to write your essay. This should be relatively easy if you took the time to mark up the sources and have a detailed outline.  Remember to add special consideration and emphasis to the commentary sections of the supporting arguments outlined in your thesis. These sentences are critical to the overall flow of the essay and where you will be explaining how the evidence supports or undermines the claim in the prompt.

Also, when referencing your sources, write the in-text citations as follows: “Source 1,” “Source 2,” “Source 3,” etc. Make sure to pay attention to which source is which in order to not incorrectly cite your sources. In-text citations will impact your score on the essay and are an integral part of the process.

After you finish writing, read through your essay for any grammatical errors or mistakes before you move onto the next essay.

Here are six must-have tips and tricks to get a good score on the synthesis essay:

  • Cite at least four sources , even though the minimum requirement is three. Remember not to plagiarize and cite everything you use in your arguments.
  • Make sure to develop a solid and clear thesis . Develop a stable stance for the claim and stick with it throughout the entire paper.
  • Don’t summarize the sources. The summary of the sources does not count as an argument. 
  • You don’t necessarily have to agree with the sources in order to cite them. Using a source to support a counterargument is still a good use of a source.
  • Cite the sources that you understand entirely . If you don’t, it could come back to bite you in the end. 
  • Use small quotes , do not quote entire paragraphs. Make sure the quote does not disrupt the flow or grammar of the sentence you write. 

ap lang synthesis essay monolingual

Discover your chances at hundreds of schools

Our free chancing engine takes into account your history, background, test scores, and extracurricular activities to show you your real chances of admission—and how to improve them.

Here is an example prompt and essay from 2019 that received 5 of the 6 total points available:

In response to our society’s increasing demand for energy, large-scale wind power has drawn attention from governments and consumers as a potential alternative to traditional materials that fuel our power grids, such as coal, oil, natural gas, water, or even newer sources such as nuclear or solar power. Yet the establishment of large-scale, commercial-grade wind farms is often the subject of controversy for a variety of reasons.

Carefully read the six sources, found on the AP English Language and Composition 2019 Exam (Question 1), including the introductory information for each source. Write an essay that synthesizes material from at least three of the sources and develops your position on the most important factors that an individual or agency should consider when deciding whether to establish a wind farm.

Source A (photo)

Source B (Layton)

Source C (Seltenrich)

Source D (Brown)

Source E (Rule)

Source F (Molla)

In your response you should do the following:

  • Respond to the prompt with a thesis presents a defensible position.
  • Select and use evidence from at least 3 of the provided sources to support your line of reasoning. Indicate clearly the sources used through direct quotation, paraphrase, or summary. Sources may be cited as Source A, Source B, etc., or by using the description in parentheses.
  • Explain how the evidence supports your line of reasoning.
  • Use appropriate grammar and punctuation in communicating your argument.

[1] The situation has been known for years, and still very little is being done: alternative power is the only way to reliably power the changing world. The draw of power coming from industry and private life is overwhelming current sources of non-renewable power, and with dwindling supplies of fossil fuels, it is merely a matter of time before coal and gas fuel plants are no longer in operation. So one viable alternative is wind power. But as with all things, there are pros and cons. The main factors for power companies to consider when building wind farms are environmental boon, aesthetic, and economic factors.

[2] The environmental benefits of using wind power are well-known and proven. Wind power is, as qualified by Source B, undeniably clean and renewable. From their production requiring very little in the way of dangerous materials to their lack of fuel, besides that which occurs naturally, wind power is by far one of the least environmentally impactful sources of power available. In addition, wind power by way of gearbox and advanced blade materials, has the highest percentage of energy retention. According to Source F, wind power retains 1,164% of the energy put into the system – meaning that it increases the energy converted from fuel (wind) to electricity 10 times! No other method of electricity production is even half that efficient. The efficiency and clean nature of wind power are important to consider, especially because they contribute back to power companies economically.

[3] Economically, wind power is both a boon and a bone to electric companies and other users. For consumers, wind power is very cheap, leading to lower bills than from any other source. Consumers also get an indirect reimbursement by way of taxes (Source D). In one Texan town, McCamey, tax revenue increased 30% from a wind farm being erected in the town. This helps to finance improvements to the town. But, there is no doubt that wind power is also hurting the power companies. Although, as renewable power goes, wind is incredibly cheap, it is still significantly more expensive than fossil fuels. So, while it is helping to cut down on emissions, it costs electric companies more than traditional fossil fuel plants. While the general economic trend is positive, there are some setbacks which must be overcome before wind power can take over as truly more effective than fossil fuels.

[4] Aesthetics may be the greatest setback for power companies. Although there may be significant economic and environmental benefit to wind power, people will always fight to preserve pure, unspoiled land. Unfortunately, not much can be done to improve the visual aesthetics of the turbines. White paint is the most common choice because it “[is] associated with cleanliness.” (Source E). But, this can make it stand out like a sore thumb, and make the gargantuan machines seem more out of place. The site can also not be altered because it affects generating capacity. Sound is almost worse of a concern because it interrupts personal productivity by interrupting people’s sleep patterns. One thing for power companies to consider is working with turbine manufacturing to make the machines less aesthetically impactful, so as to garner greater public support.

[5] As with most things, wind power has no easy answer. It is the responsibility of the companies building them to weigh the benefits and the consequences. But, by balancing economics, efficiency, and aesthetics, power companies can create a solution which balances human impact with environmental preservation.

More examples can be found here at College Board.

While AP Scores help to boost your weighted GPA, or give you the option to get college credit, AP Scores don’t have a strong effect on your admissions chances . However, colleges can still see your self-reported scores, so you might not want to automatically send scores to colleges if they are lower than a 3. That being said, admissions officers care far more about your grade in an AP class than your score on the exam.

Related CollegeVine Blog Posts

ap lang synthesis essay monolingual

How to Write the AP Lang Synthesis Essay with Example

September 5, 2023

AP Lang synthesis essay, AP Language

If you’re highly interested in learning more about writing analysis, then chances are you enrolled in AP Lang. Essentially, AP Lang is an advanced course for high schoolers that combines interest and knowledge in English with critical thinking. In the class, students learn how to analyze and synthesize a variety of texts to construct well-reasoned arguments. If you take AP Lang, then you can opt to take the AP test at the conclusion of the school year. On the exam, students write the AP Lang synthesis essay to demonstrate their learned abilities. In this article, we’ll look at what the AP Lang synthesis essay requires and show an example to provide better understanding of what to expect on the exam.

AP Lang Exam Basics

The AP Lang exam is separated into two sections. In the first section, students have one hour to answer a series of 45 multiple-choice questions. Here, about half of the questions are based on passages students read. The other half are focused on the best revision techniques. Essentially, the answers for the latter 20-22 questions are geared toward revising mock essays.

In this article, however, we’ll focus mainly on the second part of the exam: the AP Lang synthesis essay.

In this second section, students have two hours and 15 minutes to write three essays of their own design. The three open-ended questions in this section are intended to be free-response and allow for a variety of approaches. Each question is intended to allow up to 40 minutes to complete.

For the AP Lang synthesis essay, students are presented with a scenario of the College Board’s design. The scenario will provide its own thesis statement. Usually, scenarios relate to real-world problems like environmental concerns, media, or government policies.

For each scenario, students are provided with 6-7 outside sources. These sources could be in the form of an image, visual graph, or written paragraph. For written paragraphs, the sources are usually no more than 500 words.

Students are then expected to incorporate at least 3-4 of these outside sources into their essay response. The outside sources are intended to be used as supporting evidence for the student’s chosen stance or argument. Students are able to either agree with or disagree with the thesis presented in the original scenario.

AP Lang Exam – Scoring

In the second part of the AP Lang exam, students can earn a possible 6 points on each essay. 1 point is earned for the development of a thesis. Up to 4 points can be earned for evidence and commentary. The final 1 point is earned for sophistication of thought.

AP Lang Exam – Takeaways

Ultimately, the goal of the AP Lang synthesis essay is not whether the student is “right” or “wrong” in their argument. The key is that students are able to reasonably and clearly support their argument using the provided sources as evidence .

The College Board looks for your ability to identify relationships between texts , form a coherent argument , and interpret external sources .

Synthesis Essay AP Lang Examples

If you’re not sure how the questions will look on the AP Lang synthesis essay section, we’ll provide an example. After the example, we’ll break down the strengths and weaknesses of the response. That way, you’ll have a better idea of what the College Board is looking for.

Additionally, the College Board has released previous AP Lang synthesis essay examples you can review. They even have essay questions as recent as 2022 . For further support, a scoring commentary and comments from the Chief Reader are also available to view. Additionally, there are other examples you can view from earlier years .

Note: A good strategy to study for the synthesis essay AP Lang exam is to review your rhetorical devices and literary devices . Understanding how these devices function can be essential in constructing a cohesive essay.

Synthesis Essay AP Lang Examples – Sample Question

Below is a sample question from the AP Lang synthesis essay and a response to the prompt. This question was taken directly from a 2022 exam . However, the response to the question will be originally crafted for the purpose of this newsletter. As well, all supporting evidence will be originally created and does not correspond to any previous test.

The Question

Since the early 2000s, the United States government and a number of corporations have sponsored initiatives to improve education in the STEM disciplines: science, technology, engineering, and mathematics. The emphasis on STEM subjects in elementary, secondary, and higher education reflects concerns that United States students are less proficient in these areas than are students in other countries. Additionally, there is a belief that mastery in STEM fields is now essential in order to join a highly technical and specialized workforce. However, not everyone is convinced that a STEM-focused curriculum is necessary and/or effective.

In your response you should do the following:

  • Respond to the prompt with a thesis that presents a defensible
  • Select and use evidence from at least three of the provided sources to support your line of Indicate clearly the sources used through direct quotation, paraphrase, or summary. Sources may be cited as Source A, Source B, etc., or by using the description in parentheses.
  • Explain how the evidence supports your line of
  • Use appropriate grammar and punctuation in communicating your

How to Approach the Question

Maybe your first thought upon seeing this block of text is to feel overwhelmed. But don’t panic. There are effective ways to approach the question so you will be more prepared in your response.

It’s a good strategy to first isolate the thesis . What is the main idea of the text, and what is its argument?

Try it out. Reread the prompt and see if you can identify what the statement is asking you to develop an opinion on.

Think you’ve got it? In this example, we will be focusing on whether or not a STEM-focused curriculum in K-12 education is necessary and/or effective. In short, we will be arguing either for (highlighting the benefits) or against (highlighting the pitfalls) a STEM-focused curriculum.

How do we know what this statement is asking us?

Well, the statement provides a lot of background information. For example, we receive a definition of what STEM stands for. As well, we know that since 2000, there has been a greater initiative for STEM-focused classes.

When you read the prompt for the first time, it’s a great strategy to learn how to differentiate between background and contextual information from the heart of the argument .

A good way to learn how to isolate the argument is to look for transition words. Usually, these appear near the end of the question. Words like “however” and “yet” are signals that the statement is offering a differing opinion. Typically, the statement will tell you which two positions it’s offering for argument. These opinions are usually signaled by contrasting transition words.

So, now that we know what the question is asking us, what is the best way to respond?

Synthesis Essay AP Lang Examples – Sample Answer

The following is an essay response I crafted to the above question. After reading the sample, I will break down what it does well and what areas can be improved.

A STEM-focused curriculum is not as essential to providing a meaningful K-12 education. Because the majority of high school students are not proficient in STEM-focused classes, prioritizing these classes causes harm to student’s mental health and academic performance.

As seen in Source A, 60% of high school seniors in the Midwest only scored a C average in math and science-based classes (Langston). This statistic suggests that the majority of students do not resonate with STEM classes and therefore perform poorly. Earning a low score in any class does not bode well for students’ mental health.

When looking at the primary argument in Source C, it’s clear that most high schoolers prefer creative outlets to fact-based research (Kohler). Allowing students the opportunity to be more creative and initiate conversations about coursework lets students be more active in their learning. When students can discuss the nuance in their opinions, more personal growth happens. These conversations are not always easy to have in STEM-focused classes.

As well, when looking back to Source A, it’s clear that high school students in the Midwest earned higher grades, on average, in their English and art classes (Langston). This figure suggests that students perform better in these classes because they relate more to the source material. When relating to what they learn, they perform better in class.

In conclusion, STEM-focused curriculum is not as essential in K-12 education because most high school students do not relate to their STEM classes. When students do not earn satisfactory grades in these classes, it negatively affects their future college applications and job prospects.

Synthesis Essay AP Lang Examples – Answer Breakdown

So, what does this essay response get right, and where can it be improved? Let’s start with what the response does well.

First, the response establishes its thesis right away. Usually, it’s a good idea to clearly state your argument within the first paragraph. Not only is this a good practice because a reader can easily identify your stance, but also you can refer to your thesis as you write to make you stay on track.

With your thesis, it’s also a good idea to include one to two supporting sentences with the reasons why the thesis is concluded . Like in this example, I wrote that STEM-focused classes should not be prioritized because they can negatively affect both mental health and academic performance.

Another positive aspect of this response is that it is sure to not only reference but also cites its sources . It’s important that the reader understand where your information is coming from. That way, the readers can ensure you are interpreting the sources correctly.

AP Lang Synthesis Essay (Continued)

However, when rereading the instructions, it’s clear that this response fails the basic requirement of referring to at least three sources. Always make sure to reread the instructions to ensure you meet the standard requirements for incorporating source material.

Further, this AP Lang synthesis essay does not fully support its arguments . Ideas are simply stated and are not expanded upon.

For example, I mentioned a few times that earning low grades in STEM classes leads to negative mental health for high school students. However, there is no source referenced that either confirms or denies this claim. Therefore, there is no sufficient evidence to support my argument. It relies purely on inference.

Additionally, this AP Lang synthesis essay does not arrive at a sufficient level of sophistication of thought . Basically, sophistication of thought means avoiding broad generalizations and vague claims. The more specific you can be, the better your argument will sound.

Synthesis Essay AP Lang – In Conclusion

In the end, it’s always helpful to read the prompt thoroughly before writing. As well, making notes while you read could be a good strategy to pinpoint main ideas both in the prompt and the sources. That way, you can reread the material quickly. Similarly, sketching an outline may also be helpful. In addition, you should always carefully read the instructions to ensure all guidelines are followed.

As long as you avoid broad generalizations and use enough supporting evidence for your claim, you will be on the right path!

  • High School Success

' src=

Meghan Dairaghi

With a BA in English and an MFA in Creative Writing, Meghan has served as a writing tutor at the University of Missouri St. Louis and Maryville University. Additionally, Meghan has held editorial roles at River Styx and Boulevard, and was a prose reader at Farside Review . Most recently, her work has been featured in Belle Ombre , Flypaper Lit , and Mag 20/20 , among others, and she was nominated for the Mary Troy Prize in Fiction. 

  • 2-Year Colleges
  • Application Strategies
  • Best Colleges by Major
  • Best Colleges by State
  • Big Picture
  • Career & Personality Assessment
  • College Essay
  • College Search/Knowledge
  • College Success
  • Costs & Financial Aid
  • Dental School Admissions
  • Extracurricular Activities
  • Graduate School Admissions
  • High Schools
  • Law School Admissions
  • Medical School Admissions
  • Navigating the Admissions Process
  • Online Learning
  • Private High School Spotlight
  • Summer Program Spotlight
  • Summer Programs
  • Test Prep Provider Spotlight

College Transitions Sidebar Block Image

“Innovative and invaluable…use this book as your college lifeline.”

— Lynn O'Shaughnessy

Nationally Recognized College Expert

College Planning in Your Inbox

Join our information-packed monthly newsletter.

I am a... Student Student Parent Counselor Educator Other First Name Last Name Email Address Zip Code Area of Interest Business Computer Science Engineering Fine/Performing Arts Humanities Mathematics STEM Pre-Med Psychology Social Studies/Sciences Submit

logo-type-white

AP® English Language

The ultimate guide to 2016 ap® english language frqs.

  • The Albert Team
  • Last Updated On: March 1, 2022

the_ultimate_guide_to_2016 AP® English language frqs

Introduction to The Ultimate Guide to 2016 AP® English Language FRQs

Do you remember the last time you took a standardized test? If you do, you probably remember feeling nervous before the test. You were probably sitting patiently in your seat, waiting for the proctor to say go, and this feeling of overwhelming dread was settling on you.

It is entirely normal to feel nervous before a test. If you think about it, taking any test is a lot like being the first explorer to cross into uncharted territory. You have no idea what awaits you on the next page, and you are unsure of your readiness. The question is: what can we do to change it?

One of the key things you can do to eliminate that nervous energy before the test is to study. You can learn the layout of the exam, the primary skills you need to prepare and ensure you are ready for what is on the next page. That is why this guide exists, to walk you through the 2016 Free Response Questions from the AP® Language exam.

If you stick to the skills and tips given in this AP® Language FRQ guide it will go a long way towards preparing you for the test; the first step is to learn how the test breaks down.

Test Breakdown

The Free Response Questions (FRQs) are the essay portion of the AP® Language exam. The exam itself has two parts, the first is a multiple choice section, and the second is the FRQs. This guide provides an overview, strategies, and examples of the FRQs from the CollegeBoard. There is a guide to the multiple choice here .

The FRQ section has two distinct parts: 15 minutes for reading a set of texts and 120 minutes for writing three essays. The 15 minute “reading period” is designed to give you time to read through the documents for question 1 and develop a thoughtful response. Although you are advised to give each essay 40 minutes, there is no set amount of time for any of the essays. You may divide the 120 minutes however you want.

The three FRQs are each designed to test a different style of writing. The first question is always a synthesis essay – which is why they give you 15 minutes to read all of the sources you must synthesize. The second essay is rhetorical analysis, requiring you to analyze a text through your essay. The third paper is an argumentative essay.

Each essay is worth one-third of the total grade for the FRQ section, and the FRQ section is worth 55% of the total AP® test. Keep that in mind as you prepare for the exam, while the multiple-choice section is hard, the essays are worth more overall – so divide your study time evenly.

The scale for essay scores ranges from 1-9. A score of 1 being illegible or unintelligible, while a score of 9 is going to reflect the best attributes and aspects of early college level writing. You should be shooting to improve your scores to the passing range, which is 5 or above. Note that if you are struggling with the multiple choice section, a 9-9-9 on the essays can help make up for it.

The Tale of Three Essays

If you are currently taking an AP® class, you have probably experienced the style and formats of the three assignments. You may have learned about the specifics of the different types of essays in class, and you may have already found out which of the three is easiest for you. However, you must possess skill in all three to master the AP® test.

The First Essay (Synthesis)

The first essay on the test is going to be the synthesis essay . This essay can be the trickiest to master, but once you do get the hang of it, you will be one step closer to learning the others. The synthesis requires you to read six texts, which can be poems, articles, short stories, or even political cartoons.

Once you have read and analyzed the texts, you are asked to craft an argument using at least three of the documents from the set. The sources should be used to build and support your argument, and you must integrate them into a coherent whole.

On the 2016 FRQ section of the AP® exam, the synthesis essay focused on the decline of foreign language learning in English speaking countries. The complete prompt for the section is below:

Question 1

If we break down the task, it is asking you to use the six sources to create a “coherent, well-developed argument” from your own position on whether or not monolingual English speakers are at a disadvantage in today’s world. As you read this you might have some experience with the issue; perhaps you have discussed it in your foreign language or English classes. You can use that experience, but your response needs to focus on the given texts.

To find the actual documents you can go here . Taking a look at the documents will provide some context for the essay samples and their scores.

The question is scored on a scale from 1-9, with nine being the highest. Let’s take a look at some examples of student essays, along with comments from the readers – to break down the dos and don’ts of the FRQ section.

You should always strive to get the highest score possible. Writing a high scoring paper involves learning some practices that will help you write the best possible synthesis essay. Below are two examples of what you should do taken from student essays.

Connect Your Sources and Reasoning

One of the things you should do is make an explicit connection between the sources given and your reasoning. When you utilize the sources provided, make sure you are giving enough details and information to connect them to your arguments effectively. Let’s look at a student example:

Connect Your Sources and Reasoning

Examining the example above we can see how a high scoring essay integrates the sources provided with their commentary and arguments. Notice that the student blends the source materials from source A with their arguments showing the benefits of multilingual learning in the current workforce.

The student starts with the specific argument that “English speakers who can communicate with those who speak other languages are at a great advantage in their professional lives.” The student then uses the two pieces of information from source A (increasing the strength of their industry, and making themselves more employable) to provide proof for their claim.

To score high, ensure that you explicitly connect your argument in each paragraph to the sources you are using as support.

Have at Least One Source per Reason

The second thing you should do to earn a high score on the first essay is organize the sources you use into different reasons you can use to argue your claim. Let’s look at how a high-scoring student organized her ideas:

Have at Least One Source per Reason

Notice that the student was able to create three strong reasons from the information given in the sources. One of the best skills you can pick up while you practice for the AP® Lang FRQs is to choose sources that will allow you to create a coherent argument with clear reasons behind it.

There are some practices that students should avoid on FRQ 1 of the test. Students who do these things can expect to receive low scores on their essays, and if you wish to score above a five, you should avoid them at all costs.

Don’t Misread the Sources

Do not misread the information that a source is providing. Students will sometimes think that they can use a source to argue a point that it doesn’t support, those students are wrong. Every person grading the AP® Lang FRQs has read and is intimately familiar with the source material on the test, so fooling them is not an option. Let’s look at an example:

Dont Misread the Sources

This student misreads the source and tries to fit the evidence from the article in a place that it doesn’t fit. The argument that Berman makes in his article has very little to do with reassuring that “an individual’s culture is best for them” – and the evidence that the student offers doesn’t make any mention of choosing a best or better culture.

In the source, Berman is offering an alternative reason why someone might learn a language in contrast to the argument that learning languages should be entirely utilitarian. When you are writing your essay ensure that you always read the text correctly.

Don’t Give Inadequate Detail

Dont Give Inadequate Detail

If you notice in the example above the student doesn’t elaborate on the evidence in much detail. The student mentions some general ideas like that the human brain becomes “smarter, faster, and wittier…” as a result of second language acquisition, but the student doesn’t provide much explanation of how that happens, or why that even matters to their argument about the value of language acquisition.

When you are working on your test make sure that you adequately explain the evidence you use. Make explicit connections between the evidence, and your argument. Any source that is left unexplained will count against you in the long run.

AP® Readers’ Tips:

  • Read every text before you start your essay. One of the pitfalls of many students is that they do not use enough sources and try to fit them in after the fact.
  • Plan ahead. Ensure that you understand what you are going to be saying and how you will incorporate the different sources into your writing. You will need at least three sources to get above a 6, so ensure you have at least that many mapped in your plan.

The Second Essay (Rhetorical Analysis)

The second essay on the FRQ section is always a rhetorical analysis essay. This essay will focus on analyzing a text for an important aspect of the writing. In the case of the 2016 FRQ, the analysis was supposed to concentrate on rhetorical strategies:

Question 2

The prompt asks the reader to carefully read the eulogy presented by Margaret Thatcher in honor of Ronald Reagan, and write an essay analyzing the rhetorical strategies she uses in the speech. Rhetorical strategies are things like the rhetorical appeals, and rhetorical devices.

Let’s examine the do’s and don’ts for the second essay.

Utilize Specific Examples from the Text in Your Analysis

Utilize specific examples

In this high scoring essay, the student analyzes the text very effectively. The student points out how Thatcher uses precise diction, a rhetorical device, to get her point across. In this case, it is the idea that Thatcher is trying to “twist perception in her favor” by using the specific words “evil empire” when referring to Russia.

The student goes into further detail with their explanation that this choice in diction also allows Thatcher to show the “similarities in thinking” between the British and American peoples. This level of insight shows that the student has a distinct command of the material.

Using this level of detail in your analysis will ensure you score higher on the exam. It would be prudent then, to explain each piece of text you use in greater and more precise detail.

Whatever you identify in the text for your analysis, you should be able to point out precisely how it supports your main point. The more depth you can give in your analysis, the more accurate you can be with your comments, the better you will do.

Use Outside Knowledge Effectively to Strengthen Your Argument

The ability to pull in outside knowledge from your classes or books you have read will help enhance your analysis and can make it look like you know a lot more about the text itself. Let’s take a look at how a student did this on the 2016 exam:

Use Outside Knowledge

In the example above, the student can provide a more in-depth analysis of Thatcher’s words by connecting her mention of the cold war to background knowledge of what the cold war meant to many Americans at the time.

The student can flesh out the meaning behind Thatcher’s words because the student understands so much about the cold war. The student can comment on how people felt during specific events and connect that to Thatcher’s words about the actions of President Reagan.

Whenever possible, bring in background information that will help with your analysis. It might only seem like extra knowledge about the topic or author, but it could provide some insight into why they chose to write about something or show the full effect of their argument.

Some things to avoid on the literary analysis essay include specific examples of evidence from the text, and understand the rhetorical strategies that you point out.

Don’t Fail to Provide Specific and Supportive Evidence from the Text

Everything you assert about the text should be supported by specific examples. Let’s look at an example of a student essay that fails to provide specific examples:

Don’t Fail to Provide Specific

The student who wrote this essay doesn’t give their analysis the time or detail that it requires to be well done. The student glosses over the “evidence” by stating that the example of pathos (a rhetorical strategy) can be found in “lines one and two,” but they don’t ever give a specific example.

It is unfortunate that the student fails to provide a specific example, because if Thatcher is using pathos that would be a quality rhetorical strategy that the student could explain. The student’s failure to provide specific examples of text left them to only write a very general and simplistic analysis. The student doesn’t explain why Thatcher wants to be “showing her strong emotion for Ronald Reagan” or what that would do to the audience.

Overall, the student’s lack of both textual evidence and analysis led to their receiving a low score.

Don’t Try to Examine Rhetorical Strategies that You do not Understand

If you are pointing out a particular rhetorical strategy in the text, you need to make sure that you know how that strategy works. Here is an example of a student that tried to use a strategy they didn’t understand:

Dont Try to Examine Rhetorical Strategies

This student tries to point out how Thatcher uses logos (an appeal to logic) in her eulogy. However, the student says that Thatcher’s use of logos “makes her a credible source” which, while it is an indirect effect of using logos – is not the main goal when using factual or logical arguments.

If Thatcher’s desire was to make herself a credible source, she would have used ethos in her eulogy, because that is an appeal to the character of the speaker – having to do directly with credibility.

In your essay, make sure that you understand the rhetorical strategies you point out, and explain how those strengthen the author’s argument. The explicit connection between the strategy and the argument must be made to explain the effect of the strategy.

AP® Readers’ Tips

  • Pay attention to both the holistic (overall) and analytic (particular) views of the piece. You will need to understand both the text as a whole and the specific parts of the text to analyze it effectively.
  • Don’t just analyze the rhetoric used, but instead connect the rhetoric to the specific purpose that the author hopes to achieve through their speech. This rule applies to any rhetorical analysis essay.

The Third Essay (Argument)

The third and last essay of the FRQ does not respond to a particular text. Instead, the prompt focuses on crafting an argument about a particular issue. Your essay will need to argue a particular position, though most of the questions put forth by the exam will not be simple either/or questions.

Let’s look at the prompt for the third essay from 2016:

Question 3

Before we get into the do’s and don’ts of the essay, let’s talk about the particular challenge of this task. You are presented with a scenario, in this case, it deals with disobedience as a means to create social progress, and you are asked to create an argument about that issue.

For 2016, the scenario asks you to argue what part disobedience plays in social progress. You are asked to explain the extent to which Wilde’s claims are valid, and use evidence from sources of your reading, observation, and experience.

A few of the most important things you can do to ensure you score well on the essay include using an analogy to make your claim more clear and thoroughly connect the evidence you provide to your claim.

Use Analogy to Clarify Your Claim

Though it is tricky, one of the most effective strategies to show you have a command of language and understand the rudimentary skills required for crafting an argument is to use an analogy to introduce your claim. Let’s look at an essay that uses the strategy of analogy very well:

Use Analogy to Clarify Your Claim

Instead of a very definite thesis statement, this student chooses to go with an analogy in their introduction. They still provide us with a claim, “… it is through disobedience and rebellion that social progress can be made” but they don’t lay out exactly how they are going to argue it.

The analogy itself, comparing social change through rebellion to Newton’s first law of motion works to make the rest of their argument more coherent. The idea that to change the trajectory of an object it must be acted on by a force provides a great model for thinking about social change – like the essay is going to establish through the rest of their writing.

If you can utilize analogy effectively in your writing, it can work to make your writing clear to the reader. Beyond clarity, it will show you have a command of the English language and argumentative strategies.

Make Explicit Connections between Your Evidence and Claim

The best essays are going to make clear and convincing connections between the evidence they provide and the claim they are trying to support. If there are no explicit connections made between the evidence and the argument, the entire essay will be unconvincing and result in a low score.

Let’s take a look at one example of how this connection works:

Make Explicit Connections

The student does a very good job connecting their evidence (the social change brought about by non-violent protests during the civil rights movement) and their claim (that social change happens through rebellion). The connection is demonstrated very well through their explanation, and they expand on the ideas in the subsequent sentences.

The sentence, “Although they seem like small steps, these efforts were part of a larger effort to get America to realize that segregation was wrong” demonstrates the students understanding of how their evidence supports the claim of the essay. It also shows that they can break down their argument, and ensure it stands up to criticism (that the boycotts didn’t end segregation).

Overall, this student can effectively connect their evidence to the claim which resulted in a higher score.

If we take a look at the essay samples from 2016 FRQ, there are few examples that stand out as don’ts. In particular, you should avoid misunderstanding the evidence you use, and the use of unnecessary information and phrases.

Don’t Fail to Explain Your Evidence

When you are making an argument, and it is based solely on your experiences and reasoning, it can be easy to get bogged down in the details and fail to explain your evidence adequately. You need to take your time and ensure you make clear connections between your evidence and claim.

Let’s take a look at a sample from an essay that fails to explain its evidence:

Don’t Fail to Explain Your Evidence

The essay presents the novella Animal Farm as evidence. However, it fails to explain how it is evidence. In the passage above, the student says that the novel is an analogy of the Soviet Union and Communism but then says that the characters in the novel and the people in the Soviet Union caused them to disobey against their leaders.

This example is a wholly confusing paragraph. First, the question arises of who the characters and Soviet Union are causing to rebel. Second, it doesn’t explain how this shows that rebellion brings about social progress. In the novella, which many of the essay graders will have read, the animals rebel, and it eventually leads to tragedy.

This student uses a source they don’t quite understand and as a result fails to argue their point effectively. You must uses sources that you understand in your essay, and the sources you use must be directly related to your argument – unlike this student’s use of animal farm.

Don’t Use Illogical Sentences

It can be difficult to make sense when you are under time pressure, but your sentences must make sense in the context of your essay. Here is an example from a low scoring essay with sentences that don’t make sense:

Don’t Use Illogical Sentences

The student begins the paragraph by talking about how critical thinking is necessary for rebellion, which may be true. However, this has nothing to do with showing how rebellion leads to social progress.

As you read the rest of the paragraph, the student focuses more on how critical thinking is a valuable quality that shouldn’t be wasted, not on how it works to advance social change through rebellion.

If you cannot think of an argument that supports your claim, then find evidence that supports an argument or reason you wrote about earlier. Unrelated reasons and arguments do not help you score high.

  • Keep track of all parts of the prompt. One of the easiest ways to drop points is to forget to answer an important aspect of the prompt. In the case of the 2016 prompt, the essay needs to talk about the value of Wilde’s idea and argue about the role of rebellion in social progress.
  • Try to reference literary examples in your writing. Using things you have read provides a chance to go into more depth and detail about what you are writing.

General Tips from AP® Graders

  • Make a plan. One of the best things you can do for any essay you are writing under a time crunch is to make a thought-out plan. Sometimes, in the heat of writing, it is easy to forget where we are in our arguments. Having a simple outline can save you from that misfortune.
  • Answer the question in your introduction, and be direct. Directly answering the prompt is one of the easiest ways to ensure you get a higher score.
  • Clearly, indent your paragraphs, and ensure that you always have an easy to navigate structure. Topic sentences are a must, so make sure those figure into your structure.
  • Use evidence especially quotes from the texts, and explain what they mean. You need to make an explicit connection between the evidence you use, and how it supports your points.
  • Part of all great writing is variety. Vary your sentence structures, don’t make all of your sentences short or choppy, but instead try to inject some creativity into your writing. Utilize transitions, complex sentences, and elevated diction in your writing.
  • Use active voice, and make every word add to the paper as a whole. Avoid fluff; you don’t want your paper to look irrelevant because you are trying to pad your word count.

Go Forth and Conquer

Now that you better understand the expectations of the AP® Language and Composition FRQ section, you are one step closer to getting your five on the exam. Take what you have learned in this guide, and work on applying it to your writing. So, now it is time to go practice to perfection.

If you have any more tips or awesome ideas for how to study for the AP® Lang FRQ add them in the comments below.

Looking for AP® English Language practice?

Kickstart your AP® English Language prep with Albert. Start your AP® exam prep today .

Interested in a school license?​

Popular posts.

AP® Physics I score calculator

AP® Score Calculators

Simulate how different MCQ and FRQ scores translate into AP® scores

ap lang synthesis essay monolingual

AP® Review Guides

The ultimate review guides for AP® subjects to help you plan and structure your prep.

ap lang synthesis essay monolingual

Core Subject Review Guides

Review the most important topics in Physics and Algebra 1 .

ap lang synthesis essay monolingual

SAT® Score Calculator

See how scores on each section impacts your overall SAT® score

ap lang synthesis essay monolingual

ACT® Score Calculator

See how scores on each section impacts your overall ACT® score

ap lang synthesis essay monolingual

Grammar Review Hub

Comprehensive review of grammar skills

ap lang synthesis essay monolingual

AP® Posters

Download updated posters summarizing the main topics and structure for each AP® exam.

Interested in a school license?

ap lang synthesis essay monolingual

Bring Albert to your school and empower all teachers with the world's best question bank for: ➜ SAT® & ACT® ➜ AP® ➜ ELA, Math, Science, & Social Studies aligned to state standards ➜ State assessments Options for teachers, schools, and districts.

  • How to Answer the AP English Language and Composition Essay Questions 

August 1, 2021

Answering the ap english language and composition synthesis essay question, sample synthesis essay question.

SUGGESTED TIME: 

15 MINUTES FOR READING THE QUESTION AND SOURCES 

40 MINUTES FOR WRITING AN ESSAY 

Homework has always been part of going to school. In recent years, efforts to improve education have included assigning more homework to students from kindergarten to twelfth grade. Many teachers, parents, and others applaud this increase. Critics, in contrast, claim that heavier loads of homework do more harm than good, not only to children but also to their families. 

Carefully read the following six sources, including the material that introduces each source. Then, in an essay that synthesizes at least three of the sources, take a position on the claim that large amounts of homework have more negative consequences than positive ones. 

Source A (Kohn) 

Source B (Gill and Schlossman) 

Source C (Loveless) 

Source D (Chart) 

Source E (Haley) 

Source F (Chaika) 

Source G (Hanson) 

Instructions: 

  • Respond to the prompt with a thesis that may establish a line of reasoning. 
  • Provide evidence from at least three of the provided sources to support the thesis. Indicate clearly the sources used through direct quotation, paraphrase, or summary. Sources may be cited as Source A, Source B, etc., or by using the descriptions in parentheses. 
  • Explain the relationship between the evidence and the thesis. 
  • Demonstrate an understanding of the rhetorical situation. 
  • Use appropriate grammar and punctuation in communicating the argument.

Alfie Kohn, “Homework: An Unnecessary Evil?” Psychology Today , published online at www.psychologytoday.com , November 24, 2012.

The following passage is an excerpt from an article written by an author and specialist in behavior and education. His books include The Homework Myth and What Does It Mean to Be Well Educated?

At the high school level, the research supporting homework hasn’t been particularly persuasive. There does seem to be a correlation between homework and standardized test scores, but (a) it isn’t strong, meaning that homework doesn’t explain much of the variance in scores, and (b) one prominent researcher, Timothy Keith, who did find a solid correlation, returned to the topic a decade later to enter more variables into the equation simultaneously, only to discover that the improved study showed that homework had no effect after all.

. . . When homework is related to test scores, the connection tends to be strongest—or, actually, least tenuous—with math. If homework turns out to be unnecessary for students to succeed in that subject, it’s probably unnecessary everywhere. 

Along comes a new study, then, that focuses on the neighborhood where you’d be most likely to find a positive effect if one was there to be found: the effect of math and science homework on grades in high school . . . . 

This result clearly caught the researchers off-guard. Frankly, it surprised me, too. When you measure “achievement” in terms of grades, you expect to see a positive result—not because homework is academically beneficial but because the same teacher who gives the assignments evaluates the students who complete them, and the final grade is often based at least partly on whether, and to what extent, students did the homework. 

It’s important to remember that some people object to homework for reasons that aren’t related to the dispute about whether research might show that homework provides academic benefits. They argue that (a) six hours a day of academics are enough, and kids should have the chance after school to explore other interests and develop in other ways—or be able simply to relax in the same way that most adults like to relax after work; and (b) the decision about what kids do during family time should be made by families, not schools.

Brian P. Gill and Steven L. Schlossman, “My Dog Ate My Argument,” Op/Ed page of the Los Angeles Times , December 11, 2003.

The following passage is an excerpt from an opinion article written by a social scientist at the RAND Corporation and a history professor at Carnegie Mellon University. 

In our view, homework is the prime window into the school for parents to see, understand and connect with the academic mission of the teachers. It is the primary arena in which children, parents and schools interact on a daily basis. Yet it gets less systematic thought and attention than any other key component of education. Other than the admonition that kids should do more of it, we pay almost no attention to how to improve its design and content. Nor do we do much to prepare teachers to use and evaluate homework, to hold administrators accountable for monitoring the homework load or to cultivate parents’ collaboration. Homework remains an orphan child of the educational excellence movement. 

. . . After half a century of failure to increase student buy-in, it’s time to rethink how to make homework a more valued part of the pedagogic process. In addition to promoting academic achievement, homework can inculcate habits of self-discipline and independent study and can help inform parents about the educational agenda of their school. We must find ways to make homework an interesting and challenging educational experience for students, instead of the uniform, seat-bound, memorization-focused solo exercise it has been. Otherwise, all our talk about high standards and improving student achievement will run up against the same roadblock that has stymied the pursuit of educational excellence in the past.

Tom Loveless, “Do Students Have Too Much Homework?” A report for the Brown Center on Education Policy at the Brookings Institution, Washington, D.C., 2003.

The following passage is excerpted from a report on American education.

The most reliable data support the following conclusions: 1) the typical student, even in high school, does not spend more than an hour per day on homework, 2) the homework load has not changed much since the 1980s, 3) the students whose homework has increased in the past decade are those who previously had no homework and now have a small amount, 4) most parents feel the homework load is about right and, of those who would like to change it, more parents would rather see homework increased than decreased.

. . . Research shows that the relationship of homework with student achievement is positive for both middle and high school students and neutral for elementary school students. The research does not prove causality, an ever-present difficulty with research on many educational practices. High-achieving students in high school, for example, may do more homework because they enjoy studying. They take tough classes that require a lot of work. That does not necessarily mean that homework is boosting their achievement. Low-achieving students in elementary school, on the other hand, may do more homework because they are struggling to catch up. The homework is not causing their learning problems.

“Average NAEP Reading Scale Scores of Students by Age and Amount of Assigned Daily Reading Homework,” Digest of Education Statistics , National Center for Educational Statistics, 2012. 

See image below.

Brian Haley, “What Is the Value of Homework?” July 6, 2006. SearchWarp.com . Accessed August 2, 2006, http://www.searchwarp.com  

The passage that follows is adapted from an article published by a website that promotes the writing of authors in many disciplines, including education. 

Assigning homework serves various educational needs. It serves as an intellectual discipline, establishes study habits, eases time constraints on the amount of curricular material that can be covered in class, and supplements and reinforces work done in school. In addition, it fosters student initiative, independence, and responsibility, and brings home and school closer together. 

. . . Like mowing the lawn or taking out the garbage, homework seems to be a fact of life. . . . But the value of homework extends beyond school. We know that good assignments, completed successfully, can help children develop wholesome habits and attitudes. . . . It can teach children to work independently, encourage self-discipline and responsibility (assignments provide some youngsters with their first chance to manage time and meet deadlines), and encourage a love of learning. . . . Homework can help parents learn about their children’s education and communicate both with their children and the schools. 

Research in the last decade has begun to focus on the relationship between homework and student achievement and has greatly strengthened the case for homework. Although there are mixed findings about whether homework actually increases students’ academic achievement, many teachers and parents agree that homework develops students’ initiative and responsibility and fulfills the expectations of students, parents, and the public. Studies generally have found homework assignments to be most helpful if they are carefully planned by the teachers and have direct meaning to students.

Gloria Chaika, “Help! Homework Is Wrecking My Home Life,” Education World , August 8, 2000. 

The following passage is from an article for school administrators published in an online educational journal. 

“Teachers should devote energy to creating homework that is stimulating and provocative rather than banal,” says Howard Gardner of the Harvard Graduate School of Education. “And parents or mentors should go shoulder-to-shoulder with youngsters, helping to motivate them, thinking of ways in which to help them without giving the answer, and being aware of the child’s special gifts and weaknesses.” 

It sounds great, “but you need parent input for kids to perform, and with the increase in single-parent families, there’s no one at home to help,” veteran fifth-grade teacher Loretta Highfield told Education World. 

“It isn’t that the kids don’t want to do homework; the majority of my students don’t have the skills to go home and do it independently,” added Highfield, a teacher at Florida Avenue Elementary in Slidell, Louisiana. “Even young students are not getting the help at home that they used to.” 

The same seems to hold true for older children. “I have students who have been thrown out of the house or have a financial situation brought on by an ill parent,” Northshore High School (Slidell, Louisiana) teacher Kathleen Modenbach told Education World. “There are others whose after-school jobs pay for car insurance and clothes or whose involvement in extra-curricular activities, private lessons, or sports leaves little time for homework.” 

“For some students, a lot of homework can seem irrelevant,” Modenbach added. “High school students become expert at evaluating the validity of assignments and assigning priorities to them. Kids who wouldn’t dream of cheating on a test or copying a research paper think nothing of copying homework. I find students will do homework when it must be done to pass the class. Anything else is a waste of time and feeds into the vicious circle of beating the homework system.” 

Therefore, as kids deal with assigned homework in their own ways—or grow increasingly frazzled—their too-busy parents are uncertain what to do. Some, wanting their children to be academically competitive, demand extra homework, while others wonder just how much is too much.

Michael Hanson, “Analyzing ‘the Homework Gap’ Among High School Students,” Brown Center on Education Policy, 2017. 

Researchers have struggled for decades to identify a causal, or even a correctional, relationship between time spent in school and improved learning outcomes for students. Some studies have focused on the length of the school year while others have focused on hours in a day or week, and others on hours spent on homework. 

Measuring the relationship between homework and outcomes like test scores can be difficult. Researchers are primarily confounded by an inability to determine what compels students to choose homework during their time off over other activities. Are those who spend more time on homework just extra motivated? Or are they struggling students who need to work harder to keep up? What role do social expectations from parents and peers play? 

Previous studies have examined the impact of this outside time use on educational outcomes for students. A recent study from Berea College in Kentucky identified a causal relationship between hours spent studying and a student’s academic performance through an interesting measure. The researchers took advantage of randomly assigned college roommates, paying attention to those who came to campus with smart phones packed with video games. They hypothesized students randomly assigned to a roommate without much interest in video games would study more, since all other factors remained equal. That hypothesis held up, and that group also received higher grades, demonstrating a causal relationship. 

Other research has relied on data collected through the American Time Use Survey, a study of how Americans spend their time, and [has] shown the existence of a gender gap and a parental-education gap in homework time. Other studies have looked at the relationship between holding a job and students’ time use in discretionary activities, like sleep, media consumption, and time spent on homework.

SOURCE D (referenced above)

The table below has been adapted from research conducted by the National Association of Educational Progress, the nation’s largest testing agency responsible for assessing what America’s K–12 students know and can do in various subjects.

ap-english-lang-and-comp-synthesis-essay-source

How to Answer the Synthesis Essay Question

Homework. Now, there’s a topic that you must know something about. Being a seasoned doer of homework, you’re probably bursting with ideas on the pros and cons of the stuff and could probably argue brilliantly for or against homework, or come down somewhere between the two poles. Regardless of where you stand, you’re not apt to find yourself short of ideas on the issue. In fact, you may be overloaded and find yourself sifting out only the best arguments among many to include in an essay on the subject.

But beware. This essay assignment is not intended simply to give you a chance to vent about homework. Although your biases will no doubt shape your argument, you mustn’t rely solely on your personal experience and observations. This, after all, is what the AP people call a “ synthesis essay ,” a label that you’ve got to take seriously.

AP English Language and Composition Synthesis Essay Step #1: Cite Sources

Stylistically, it may serve you well to use phrases like “According to Source C, . . .” or “In Loveless’s opinion . . .”, or “A study of students’ reading scores (Source D) shows that . . . ,” etc. Or you can simply cite your sources with parenthetical references—(Source A, Source B)—in your text. Another approach is to name the author or even the title of the sources, but writing out lengthy titles uses up precious time. AP essay readers will look for citations and will penalize essays that contain fewer than three. At the same time, however, you won’t earn extra credit for citing more than three. 

Whether or not you agree with the premise that “large amounts of homework have more negative effects than positive ones,” your task is to write an argument that defends your point of view. Because a researched argument is meant to sway readers whose views may be contrary to yours, you need to gather compelling evidence in support of your position. 

Let’s say that you think homework is generally good for you and the more you get, the better. Right off the bat, then, you have a main idea, or thesis, for your essay. But even if you know immediately where you stand on the issue, take the time to read all the sources carefully, underscoring or circling those ideas you might consider mentioning in your essay. It’s good to read the material with which you don’t agree, too, because in making your case, you can bolster your argument by refuting and revealing the weaknesses in what you’d expect your opponent to say.

AP English Language and Composition Synthesis Essay Step #2: Support Your Position

In building a convincing case, it often pays to gather at least three compelling reasons  to support your position. Although AP students ought not be constrained by the familiar “five-­paragraph” essay, you won’t go wrong following its structure: an introduction, three paragraphs of development, and a conclusion. Why three paragraphs of development? Mainly because three is a number that works. If you can come up with three different arguments, you appear to speak with the voice of authority. One paragraph is too simple. Two is better but still shallow. Three is thoughtful. It suggests depth and insight. Psychologically, three also creates a sense of wholeness for the reader, like the beginning, middle, and end of a story. (Incidentally, it’s no accident that the number three recurs in all literature, from Goldilocks and the Three Bears to the Bible.) Use the sources to bolster your arguments for or against large amounts of homework. But you needn’t depend totally on the sources. In fact, AP readers are likely to look kindly on your own original ideas, provided they are relevant to the issue, clearly expressed, and well-­ developed. On the positive side, you might pick out such ideas as: 

  • Homework permits parents to participate with teachers in the education of their children. (Source B)
  •  “[T]he relationship between the amount of reading homework and performance on reading tests is especially positive for high school students.” (Source D) 
  •  Homework fosters the development of individual initiative and effective study habits. (Source B) 
  • Homework provides opportunities for low-achieving students to catch up. (Source C) 
  • Homework leads to a lifelong love of learning. (Source F) 
  • Homework generally contributes to higher grades, and higher grades can lead to admission to higher-quality colleges. (Source G) 

Or, if you have an unfavorable view of homework, the following ideas can be used to support your argument: 

  • Years of educational research have found only a weak correlation between homework and student achievement. (Source A) 
  • Large amounts of homework can keep a student from pursuing worthwhile personal interests. (Source C) 
  • Homework assigned during vacations is counterproductive; it turns kids away from the joys of learning and deprives them of reading for pleasure. (Source E) 
  • More homework does not necessarily lead to better grades. (Source E) 

The given sources either support or decry homework. A middle-of-the-road position may be difficult to defend unless you build a case by refuting arguments presented on both sides of the issue. Source F, which argues against homework, for example, quotes an apparently frustrated teacher: “It isn’t that kids don’t want to do homework; the majority of my students don’t have the skills to go home and do it independently.” 

Because the word “majority” can mean almost all or just over half , the teacher appears to have overlooked the fact that some students can be counted on to work on their own. By generalizing about all students, the teacher in effect deprives some of her kids the opportunity to learn at home. An essay that argues neither for nor against homework might emphasize that universal policies regarding homework don’t work. In other words, when it comes to education, one size cannot fit all.

AP English Language and Composition Synthesis Essay Step #3: Determine Order

Once you’ve collected your ideas for or against the issue, stop for a moment to figure out which idea to put first, which to put second, and so on. Order is important. The best order is the clearest order, the arrangement that readers can follow with the least effort. No plan is superior to another, provided you have a valid reason for using it. The plan least likely to succeed is the aimless one, the one in which you state and develop ideas in random order as they happened to come to mind. It’s better by far to rank your ideas in order of importance by deciding which provides the strongest support for your thesis. Although your best argument may be listed first in your notes, save it for last on the essay. Giving it away at the start is self- defeating because everything that follows will be anticlimactic. An excellent way to arrange your ideas is to lead with your second best, save your best for the end, and sandwich the others in between. This structure recognizes that the end and the beginning of an essay are its most critical parts. A good opening draws the reader in and creates an all-important first impression, but a memorable ending, coming last, is what readers have fresh in their minds when they assign you a grade. But, as always, don’t just follow these guidelines slavishly. If you can justify another organization, by all means use it. 

AP exam readers won’t judge your essay based on the opinion you express. Even if they disagree with you, they are obliged to ignore their own biases and grade you according to the criteria of good writing. They may think that your view is off the wall, but a cogent, forceful essay that smoothly integrates the sources and demonstrates mastery of argumentation will merit a high score.

Answering the AP English Language and Composition Rhetorical Analysis Essay Question

Sample rhetorical analysis question.

SUGGESTED TIME: 40 MINUTES

Read the following passage published back in 1967 by The New York Times. Then write an essay in which you analyze how the structure of the passage and the use of language help convey the writer’s views.

Instructions:

  • Respond to the prompt with a thesis that may establish a line of reasoning.
  • Select and use evidence to develop and support the line of reasoning.
  • Explain the relationship between the evidence and the thesis.
  • Demonstrate an understanding of the rhetorical situation.

Rhetorical Analysis Passage

Americans and Western Europeans, in their sensitivity to lingering problems around them, tend to make science and progress their scapegoats. There is a belief that progress has precipitated widespread unhappiness, anxieties, and other social and emotional problems. Science is viewed as a cold mechanical discipline having nothing to do with human warmth and the human spirit. 

But to many of us from the nonscientific East, science does not have such repugnant associations. We are not afraid of it, nor are we disappointed by it. We know all too painfully that our social and emotional problems festered long before the age of technology. To us, science is warm and reassuring. It promises hope. It is helping us at long last gain some control over our persecutory environments, alleviating age-old problems—not only physical but also, and especially, problems of the spirit.

Shiraz, for example, a city in southern Iran, has long been renowned for its rose gardens and nightingales; its poets, Sadi and Hafiz; and its mystical, ascetic philosophy, Sufism. Much poetry has been written in glorification of the spiritual attributes of this oasis city. And to be sure, Shiraz is a green, picturesque town, with a quaint bazaar and refreshing gardens. But in this “romantic” city thousands of emotionally disturbed and mentally retarded men, women, and children were, until recently, kept in chains in stifling prison cells and lunatic asylums. 

Every now and again, some were dragged, screaming and pleading, to a courtyard and flogged for not behaving “normally.” But for the most part, they were made to sit against damp walls, their hands and feet locked in chains, and thus immobilized, without even a modicum of affection from their helpless families and friends, they sat for weeks and months and years—often all their lives. Pictures of these wretched men, women, and children can still be seen in this “city of poetry,” this “city with a spiritual way of life.” 

It was only recently that a wealthy young Shirazi who, against the admonitions of his family, had studied psychology at the University of Teheran and foreign universities, returned to Shiraz and after considerable struggle with city officials succeeded in opening a psychiatric clinic, the first in those regions. After still more struggle, he arranged to have the emotionally disturbed and the mentally retarded transferred from prison to their homes, to hospitals, and to his clinic, where he and his staff now attend them. 

They are fortunate. All over Asia and other backward areas, emotionally disturbed men and women are still incarcerated in these medieval dungeons called lunatic asylums. The cruel rejection and punishment are intended to teach them a lesson or help exorcise evil spirits. 

The West, still bogged down in its ridiculous romanticism, would like to believe that emotional disturbances, dope addiction, delinquency are all modern problems brought on by technological progress, and that backward societies are too spiritual and beautiful to need the ministrations of science. But while the West can perhaps afford to think this way, the people of backward lands cannot. . . . 

. . .The obstacles are awesome, the inertia too entrenched, the people’s suffering too anguished, their impatience too eruptive. Moreover, the total cultural reorganizations such as Asia and Africa are undergoing inevitably engender their own temporary dislocations and confusions. But their goals, the direction, remain constant. We are on the move, however awkwardly at first, to a saner, better world.

How to Answer the Rhetorical Analysis Question

Go back to the original question, which asks you to analyze two features of the passage: (1) its structure, or organization, and (2) its language. The first aspect is fairly specific. As you read the passage, you need to observe what the author discusses first, second, third, and so on. Your essay should explain not only the order of ideas but the reasons the author may have chosen that order. 

The second part of the question is more general. It invites you to analyze the use of language, which may include the author’s choice of words (diction), syntax (word order), figures of speech, use of evidence (such as statistics or logical reasoning), sentence structure, rhythm, sound, tone, or just about any other characteristics of style and rhetoric you choose.

Although the question directs you to write about two different aspects of the passage, the essay itself should be unified. That is, a good essay should not consist of, say, two disparate paragraphs, one exclusively devoted to structure and another to language. Rather, the essay should include material that shows the interrelationship of structure and language in the passage and how those elements contribute to the meaning and effect of the passage. This might be covered in a separate paragraph, or it could be woven into the overall fabric of the essay. 

Before you begin to write , read the passage at least twice: once for an overview and once as you write your analysis. You may notice early on that the opening paragraph contains generalizations about Westerners’ concepts of science and progress. Then the author contrasts the Western view of science and progress with the Eastern view. Immediately, you see that the author, by using the first-person pronoun (as in “many of us”) is speaking from the perspective of an Easterner. Consequently, his discussion of Eastern views is apt to come across as more well-informed, more authoritative, perhaps more personal. 

To support his position, the author gives an extended example—the city of Shiraz—to illustrate just how different the East is from the West. The description and vivid images of Shiraz memorably convey the idea that the “spiritual way of life” has a side to it that many Westerners don’t know about. This is the heart of the passage. The use of quotation marks around “romantic” and “city of poetry” is meant to point out the discrepancy between the idealized and real versions of Shiraz. 

Nearing the end, the author reiterates his initial contrast between West and East, with emphasis on the East. The last paragraph offers a generalized statement about conditions in Asia and Africa, reminding the reader of the contrast made at the very beginning of the passage. Tying the end to the beginning of the passage creates a sense of unity—a desirable feature in any piece of writing.

Answering the AP English Language and Composition Argument Essay Question 

The third essay on the exam requires you to respond to an idea contained in a short statement or paragraph. Your response must be written as an argument that either supports or refutes a writer’s views on a particular subject. Or, if you prefer not to take an either/or position, you can adopt a stance somewhere in between the two.

Writing a persuasive essay involves more than simply expressing your opinion on an issue. The validity of your position must be based on sound evidence. Passion alone won’t do it. You need to corral evidence from your experience, reading, studies, and observation in order to prove that your opinion has merit. 

To argue on behalf of your position, find at least two (three is even better) distinct arguments to support it. It helps, too, to develop a counterargument—an argument most likely to be used by someone who opposes your views—that you can refute in order to persuade readers that you are right and your opponent is not. 

Because topics for AP persuasive essays are unpredictable, it makes sense to arm yourself with a ready-to-use essay-writing strategy—one that, regardless of the topic, lays out the steps to take during the approximately forty minutes it takes to complete the essay. Chances are that you’ve written reams of essays during your school career. Over the years, you may have developed a method for writing blue-ribbon essays. But in case you haven’t, here is a list of steps you can count on. Follow them while you write essays for practice. Then, based on the results you get, amend the list in ways that enable you to write the best essays you can.

  • Read and analyze the prompt.
  • Jot down ideas that might be used to argue both sides of the issue.
  • Review the ideas and choose a position on the issue.
  • Articulate a main idea, or thesis, for your essay.
  • Arrange supporting ideas purposefully—not simply in the order they occurred to you.
  • Introduce the main idea of your essay.
  • Develop unified paragraphs in support of your main idea.
  • Devote at least part of your essay to refute an argument likely to be used by someone whose opinion differs from yours.
  • Choose words and structure sentences that concisely convey your thoughts.
  • Write a memorable conclusion but not a brief summary of your essay.
  • Edit your essay for clarity, interest, and correctness.

Experience shows that these steps do not need be taken in the order presented, nor is each step discrete. Rather, they often overlap and blend into each other. While composing your essay, for example, you may also be revising and proofreading. Late in the process, you may weave new ideas into your text or shift the location of ideas. In short, no step really ends until the final period is put into place or the AP proctor calls “Time!” We can't tell you exactly how much of the suggested 40-minute writing period to devote to each step. A plan that works for other students may not work for you. In general, however, you won’t go wrong by devoting more than half the time—about 25–30 minutes—to composing an essay and no more than 5–10 minutes planning and polishing it. By now you may have noticed that the basic process of writing a persuasive essay hardly differs at all from that used in writing synthesis or analytical essays. All three require you to read the prompt over and over until you are absolutely sure of what it says and what you are expected to do. The prompt may not interest you right away, but if you really concentrate on the issue, you may soon be bursting with ideas for your essay.

Sample Argument Essay Question

SUGGESTED TIME: 40 MIINUTES

The following paragraph is adapted from Mirror for Man, a book written by anthropologist Clyde Kluckhorn in the middle of the twentieth century. Read the passage carefully. Then, write an essay that examines the extent to which the author’s characterization of the United States holds true today. Use appropriate evidence to support your argument.

Technology is valued as the very basis of the capitalistic system. Possession of gad- gets is esteemed as a mark of success to the extent that persons are judged not by the integrity of their characters or by the originality of their minds but by what they seem to be—so far as can be measured by their wealth or by the variety and material goods which they display. “Success” is measured by their investments, homes, and lifestyles— not by their number of mistresses as in some cultures.

How to Answer the Argument Question

Whether you agree, disagree, or have mixed views on the content of the passage, your job is to write a convincing argument that expresses your opinion. Initially, the word argument may suggest conflict or confrontation. But rest assured that your essay need not be combative. Rather, make it a calmly-reasoned explanation of your opinion on a debatable subject. Your goal is to persuade the reader that your opinion, supported by examples, facts, and other appropriate evidence, is correct. 

If you have strong feelings about the topic, of course you should state them in your essay. But express them in calm, rational language. Be mindful that the essay should not be an emotional rant for or against the issue. 

Consider first whether you agree with Kluckhorn’s definition of “success.” Is it, as Kluckhorn asserts, measured by income and material possessions? Or do you think that a more accurate standard of success in today’s America should be determined by less tangible criteria—things such as happiness or self-respect? Or do you stand somewhere in between those two extremes? 

The actual position you take on the issue is less crucial than your ability to support it fully by drawing from your knowledge, background, experience, or observation. Regardless of your position, be sure to include more than one example. An argument that relies on a single example, however compelling, will fall flat. 

In the prompt, Kluckhorn’s notion of success seems to refer broadly to American society. Resist responding in kind. That is, a short essay shouldn’t focus on the whole of society but only on an identifiable segment—perhaps college-educated professionals or urban, blue-collar Americans. The point is that a narrowly focused essay on a limited topic will always turn out better than one that tries to cover too much ground in just a few paragraphs.

AP Biology Resources

  • About the AP Biology Exam
  • Top AP Biology Exam Strategies
  • Top 5 Study Topics and Tips for the AP Biology Exam
  • AP Biology Short Free-Response Questions
  • AP Biology Long Free-Response Questions

AP Psychology Resources

  • What’s Tested on the AP Psychology Exam?
  • Top 5 Study Tips for the AP Psychology Exam
  • AP Psychology Key Terms
  • Top AP Psychology Exam Multiple-Choice Question Tips
  • Top AP Psychology Exam Free Response Questions Tips
  • AP Psychology Sample Free Response Question

AP English Language and Composition Resources

  • What’s Tested on the AP English Language and Composition Exam?
  • Top 5 Tips for the AP English Language and Composition Exam
  • Top Reading Techniques for the AP English Language and Composition Exam
  • AP English Language and Composition Exam Sample Essay Questions
  • AP English Language and Composition Exam Multiple-Choice Questions

AP Human Geography Resources

  • What’s Tested On the AP Human Geography Exam?
  • AP Human Geography FAQs
  • AP Human Geography Question Types and Strategies
  • Top 5 Study Tips for the AP Human Geography Exam

FOLLOW ALONG ON SOCIAL

Calculate for all schools

Your chance of acceptance, your chancing factors, extracurriculars, perfecting the synthesis essay for ap lang.

Hi there! I'm preparing for the AP Lang exam, specifically working on synthesis essays. Are there any sample essays or best practices to construct an effective argument while incorporating various viewpoints?

Hi! It's great that you're working on improving your synthesis essay skills for the AP Lang exam. Here are some tips and best practices to construct an effective argument while incorporating various viewpoints:

1. Understand the prompt: Make sure to read the prompt carefully and understand what is being asked. It might help to rephrase it in your own words to ensure you've fully grasped the concept.

2. Develop a clear thesis: Your thesis statement should present a clear and concise argument that addresses the prompt. It should be disputable and take a definitive stance - the key is to show that you have a clear opinion on the issue.

3. Create an outline: Before you start writing, creating an outline can be helpful for organizing your thoughts and planning your essay. This will help you structure your essay and ensure that you address all aspects of the prompt, as well as prevent you from losing your train of thought halfway through.

4. Use your sources effectively: Make sure to incorporate at least three of the provided sources in your essay, and use them to support your argument. Integrate your sources by quoting, paraphrasing, or summarizing, but make sure to cite them properly, whether it's through in-text citations or footnotes.

5. Acknowledge counterarguments: By acknowledging and addressing counterarguments, you strengthen your own argument, as considering multiple perspectives helps underscore that yours is in fact the best one out there. Be respectful and fair when addressing opposing viewpoints, and use your sources to refute them effectively.

6. Organize your essay: Use clear transitions between paragraphs and ideas to guide the reader through your argument. Arrange your paragraphs logically, making sure each one presents a distinct point or example that supports your thesis.

7. Strong conclusion: The conclusion should summarize your argument, restate your thesis, and leave the reader with a memorable impression of your essay.

To find sample essays, you can refer to the College Board's official AP Lang exam resources, which include released student sample responses from past exams. Additionally, practicing writing synthesis essays yourself using past exam prompts is an effective way to prepare for the AP Lang exam. As you practice, try to address different perspectives, incorporate sources effectively, and maintain a clear and logical organization in your essay.

Good luck with your studying!

About CollegeVine’s Expert FAQ

CollegeVine’s Q&A seeks to offer informed perspectives on commonly asked admissions questions. Every answer is refined and validated by our team of admissions experts to ensure it resonates with trusted knowledge in the field.

IMAGES

  1. How to write a good synthesis essay ap lang

    ap lang synthesis essay monolingual

  2. AmStud

    ap lang synthesis essay monolingual

  3. How To Write A Good Ap English Synthesis Essay

    ap lang synthesis essay monolingual

  4. Synthesis Essay: Basic Guide on Writing a Good Essay

    ap lang synthesis essay monolingual

  5. How to Write an AP Lang Synthesis Essay: A Complete Guide

    ap lang synthesis essay monolingual

  6. How to Write a Synthesis Essay: Examples, Topics, & Synthesis Essay Outline

    ap lang synthesis essay monolingual

VIDEO

  1. Toward effective electrocatalytic C–N coupling for the synthesis of organic nitrogeno...

  2. League Champions and their cooler versions

  3. Synthesis essay PROCESS

  4. Writing a Synthesis Essay, Composite Summary and Referencing During Examination/ UGRC210 PastQuo2024

  5. Synthesis Activities

  6. Ab tbse sir uthe nahi hai 😂🤣#chetannn026 #funny #chetannn #comedy

COMMENTS

  1. Synthesis Essay Materials

    The two synthesis essay questions below are examples of the question type that has been one of the three free-response questions on the AP English Language and Composition Exam as of the May 2007 exam. The synthesis question asks students to synthesize information from a variety of sources to inform their own discussion of a topic. Students are given a 15-minute reading period to accommodate ...

  2. How to Write a Perfect Synthesis Essay for the AP Language Exam

    Paragraph 1: The prompt presents and briefly explains the topic that you'll be writing your synthesis essay about. That topic is the concept of eminent domain. Paragraph 2: The prompt presents a specific claim about the concept of eminent domain in this paragraph: Eminent domain is productive and beneficial.This paragraph instructs you to decide whether you want to defend, challenge, or ...

  3. PDF AP ENGLISH LANGUAGE AND COMPOSITION 2016 SCORING GUIDELINES

    9 - Essays earning a score of 9 meet the criteria for the score of 8 and, in addition, are especially sophisticated in their argument, thorough in development, or impressive in their control of language. 8 - Effective. Essays earning a score of 8 effectively argue a position on whether monolingual English speakers are at a disadvantage today.

  4. AP English Language and Composition Past Exam Questions

    Download free-response questions from past exams along with scoring guidelines, sample responses from exam takers, and scoring distributions. If you are using assistive technology and need help accessing these PDFs in another format, contact Services for Students with Disabilities at 212-713-8333 or by email at [email protected]. Note ...

  5. How to Write the AP Lang Synthesis Essay + Example

    The AP Lang synthesis essay is the first of three essays included in the Free Response section of the AP Lang exam. The exam presents 6-7 sources that are organized around a specific topic, with two of those sources purely visual, including a single quantitative source (like a graph or pie chart). The remaining 4-5 sources are text-based ...

  6. PDF ENGLISH LANGUAGE AND COMPOSITION

    (This question counts for one-third of the total essay section score.) Over the past several decades, the English language has become increasingly globalized, and it is now seen by many as the dominant language in international finance, science, and politics. Concurrent with the worldwide spread of English is

  7. PDF AP English Language and Composition

    In your response you should do the following: Respond to the prompt with a thesis that presents a defensible position. Select and use evidence from at least three of the provided sources to support your line of reasoning. Indicate clearly the sources used through direct quotation, paraphrase, or summary.

  8. How to Write the AP Lang Synthesis Essay: Write the Essay

    For everything you need to know about the AP English Language Exam, check out our ultimate guide: https://marcolearning.com/guide-to-the-ap-english-language-...

  9. How to Write the AP Lang Synthesis Essay with Example

    Synthesis Essay AP Lang Examples - Sample Answer. The following is an essay response I crafted to the above question. After reading the sample, I will break down what it does well and what areas can be improved. A STEM-focused curriculum is not as essential to providing a meaningful K-12 education. Because the majority of high school students ...

  10. PDF AP English Language and Composition Synthesis Essay Sample Student

    AP®English Language and Composition Synthesis Essay Sample Student Responses. The College Board: Connecting Students to College Success. The College Board is a not-for-profit membership association whose mission is to connect students to college success and opportunity. Founded in 1900, the association is composed of more than 5,000 schools ...

  11. PDF AP® English Language and Composition 2009 Scoring Guidelines Form B

    The College Board. The College Board is a not-for-profit membership association whose mission is to connect students to college success and opportunity. Founded in 1900, the association is composed of more than 5,600 schools, colleges, universities and other educational organizations. Each year, the College Board serves seven million students ...

  12. PDF AP Scoring Rubric for Question 1: Synthesis Essay

    AP ® English Language and Composition. Scoring Rubric for Question 1: Synthesis Essay. 0 POINTS. 1 POINT: For any of the following: No defensible thesis Simple restatement of prompt only Summary of topic with no clear claim States an apparent fact rather than a defensible claim Off-topic

  13. The Ultimate Guide to 2016 AP® English Language FRQs

    Test Breakdown. The Free Response Questions (FRQs) are the essay portion of the AP® Language exam. The exam itself has two parts, the first is a multiple choice section, and the second is the FRQs. This guide provides an overview, strategies, and examples of the FRQs from the CollegeBoard. There is a guide to the multiple choice here.

  14. PDF AP English Language and Composition 2016 Free-Response Questions

    It is suggested that you spend 15 minutes reading the question, analyzing and evaluating the sources, and 40 minutes writing your response. Note: You may begin writing your response before the reading period is over. (This question counts for one-third of the total essay section score.) Over the past several decades, the English language has ...

  15. PDF Samples are typed exactly as students wrote them.

    2016 Question 1 Language Student Samples Samples are typed exactly as students wrote them. *For the purposes of scoring, synthesis means using sources to develop a position and citing them accurately. Sample L (1) Monolingual English speakers are really not at a disadvantage in the United States because we are close to the border.

  16. Writing a synthesis essay for AP Lang

    Sure, a synthesis essay is a type of essay that requires you to use multiple sources to create an argument. In an AP Lang synthesis essay, you'll typically be provided with the sources and will need to analyze them, identify the main ideas, and then connect those ideas to your central argument or thesis. Here's a step-by-step breakdown of how to write a strong synthesis essay for AP Lang: 1.

  17. AP English Language and Composition Questions

    AP English Language and Composition Synthesis Essay Step #1: Cite Sources. Stylistically, it may serve you well to use phrases like "According to Source C, . . ." or "In Loveless's opinion . . .", or "A study of students' reading scores (Source D) shows that . . . ," etc. Or you can simply cite your sources with parenthetical ...

  18. PDF ENGLISH LANGUAGE AND COMPOSITION

    The chart below tracks the average inflation-adjusted hourly wage for young college graduates with no advanced degree from 1979 to 2010. After gains in the 1980s and particularly in the 1990s, hourly wages for young college-educated men in 2000 were $22.75, but that dropped by almost a full dollar to $21.77 by 2010.

  19. Perfecting the Synthesis Essay for AP Lang?

    Hi! It's great that you're working on improving your synthesis essay skills for the AP Lang exam. Here are some tips and best practices to construct an effective argument while incorporating various viewpoints: 1. Understand the prompt: Make sure to read the prompt carefully and understand what is being asked. It might help to rephrase it in your own words to ensure you've fully grasped the ...

  20. PDF AP English Language and Composition

    AP® English Language and Composition 2022 Scoring Guidelines . Synthesis Essay 6 points . Since the early 2000s, the United States government and a number of corporations have sponsored initiatives to improve education in the STEM disciplines: science, technology, engineering, and mathematics.

  21. PDF Links for the Lesson

    the context, figurative language and other poetic techniques for the literature to be compared. Students will show mastery of the standards at the end of the lesson through a Socratic discussion, TTPCAST, AP Multiple Choice and written timed essay scored on an AP style rubric. Essential Questions 1.

  22. PDF Ap English Language & Composition

    • Essays are worth 55% of the score. Stable Argument Wording: [Topical discussion/introduction/ quotation(s) and background]. Write an essay that argues your . position on [specific subject from . the introductions]. Free Response Rubrics . Essays are graded on a scale of 1-6; the rubrics contain three rows: • Row A: Thesis (0-1 pt)

  23. How would one write a synthesis essay on space exploration for AP

    Indeed, the 2009 AP English Language and Composition synthesis essay prompt is about space exploration: "Explorers and tales of explorations tend to capture the human imagination. However, such ...